You are on page 1of 81

Career Development Centre

1
The Foreword
Aptitude Development has become paramount for securing better jobs these days. An aptitude
test makes it possible for recruiters to determine how one completes some job and how his
persona fits well within the work environment. Most of the world’s recruitment companies as a
part of their recruitment process rely on pre-employment tests, such as aptitude and
personality tests, as the most effective method to measure one’s ‘fit’, or match, for a position
applied for. As such Aptitude tests have become an important and integral part of the overall
interview process.

Periodic modifications have been made in Aptitude tests patterns designed for recruitments, in
recent years. Aptitude Refresher intends to be an absolute guide to the students who still
struggle to grasp concepts & lack time track while attempting questions. The booklet elaborates
all important topics & their concepts with solved examples.

What’s new in Aptitude Refresher!


 Fully Solved Questions
 Detailed Explanation of Important Concepts with Examples
 Short Practice Exercises for Each Concept
 Time Bound Review Exercises
 Short Tricks & Tips to Solve Questions

How to Go Ahead with this Booklet


 Step 1 – Attempt linked topics together. Example – Percentages and Profit, Loss &
Discount share similar concepts.
 Step 2 – Start with the concepts. Pay close attention to tricks & shortcuts explained. Do
examples and practice exercises and then move ahead to next concept.
 Step 3 – Do time bound practice while attempting Practice & Review Exercises.
 Step 4 – After completion of each chapter, practice more questions. You may refer the
handouts given in classroom training.
 Step 5 – Practice is the key! You might miss on time tracking & short tricks if given gap.

All the best!

2
Booklet Index

S.No. Topic Page


1 Percentage 4-8
2 Profit, Loss & Discount 9 – 14
3 Review Exercise 1 – Simple & Compound Interest 15
4 Review Exercise 2 – Number Series 16
5 Review Exercise 3 – Letter Series 17
6 Ratio & Proportion 18-21
7 Alligations 22-25
8 Mixtures 26-29
9 Partnership 30-31
10 Review Exercise 4 – Blood Relations 32
11 Review Exercise 5 – Direction Sense Test 33
12 Review Exercise 6 – Coding Decoding 34
13 Review Exercise 7 – Average 35
14 Time and Work 36-41
15 Pipes and Cisterns 42-43
16 Review Exercise 8 – Seating Arrangement 44
17 Review Exercise 9 – Ranking 45
18 Review Exercise 10 – Ages 46
19 Number System 47-58
20 Review Exercise 11 – Non Verbal Reasoning 59-62
21 Time, Speed and Distance 63-69
22 Review Exercise 12 - Data Sufficiency 70
23 Permutation & Combination 71-74
24 Probability 75-78
25 Review Exercise 13 - Mensuration 79-80
26 Review Exercise 14 - Set Theory 81

3
PERCENTAGES

When we say "Percent" we are saying "per 100"

One percent (1%) means 1 per 100.

Remember: x% of y = y% of x
Example: Find 8% of 50.

8% of 50 is the same as 50% of 8


And 50% of 8 is 4
So, 8% of 50 is 4

Decimals, Fractions & Percentages are just different ways of showing the same value:

A Half can be written as:

4
Common Fractions with Decimal and Percent Equivalents

Here is a table of commonly used values shown in Percent, Decimal and Fraction form:

Fraction Decimal Percent

1/2 0.5 50%


1/3 0.333… 33.333…%
2/3 0.666… 66.666…%
1/4 0.25 25%
3/4 0.75 75%
1/5 0.2 20%
2/5 0.4 40%
3/5 0.6 60%
4/5 0.8 80%
1/6 0.1666… 16.666…%
5/6 0.8333… 83.333…%
1/8 0.125 12.50%
3/8 0.375 37.50%
5/8 0.625 62.50%
7/8 0.875 87.50%
1/9 0.111… 11.111…%
2/9 0.222… 22.222…%

4/9 0.444… 44.444…%

5/9 0.555… 55.555…%

7/9 0.777… 77.777…%

8/9 0.888… 88.888…%

1/10 0.1 10%


1/12 0.08333… 8.333…%

1/16 0.0625 6.25%


1/32 0.03125 3.13%

5
LET’S PRACTICE THE CONVERSIONS NOW -

A. FROM PERCENT TO DECIMAL:

To convert from percent to decimal : divide by 100, and remove the "%" sign.

The easiest way to divide by 100 is to move the decimal point 2 places to the left:

B. FROM DECIMAL TO PERCENT:

To convert from decimal to percent : multiply by 100, and add a "%" sign.

The easiest way to multiply by 100 is to move the decimal point 2 places to the right:

Or you can simply multiply 0.125 with 100 and add the % sign to get 12.5%.

C. FROM FRACTION TO DECIMAL:

The easiest way to convert a fraction to a decimal is to divide the top number by the bottom number
(divide the numerator by the denominator in mathematical language)

Example: Convert 2/5 to a decimal.


Divide 2 by 5: 2 ÷ 5 = 0.4
Answer: 2/5 = 0.4

D. FROM DECIMAL TO FRACTION:

To convert a decimal to a fraction , remove the decimal by adding the denominator with appropriate
number of zeroes and then simplify the fraction.

6
Example: To convert 0.75 to a fraction
Remove the decimal => 0.75 = 75/100
Simplify the fraction => 75/100 = 3/4
Answer: 2/5 = 0.4

E. FROM FRACTION TO PERCENTAGE:

The easiest way to convert a fraction to a percentage is to multiply the fraction by 100 and reduce it to
decimal form and add the "%" sign.

Example: Convert 3/8 to a percentage


Multiply 3/8 by 100: 37.5
Add the "%" sign: 37.5%
Answer: 3/8 = 37.5%

F. FROM PERCENTAGE TO FRACTION:

To convert a percentage to a fraction , first convert to a decimal (divide by 100), then use the steps for
converting decimal to fractions (like above).

ATTENTION PLEASE!!!

REMEMBER THAT THE BASE TO BE TAKEN IS ALWAYS THE ORIGINAL QUANTITY!!!

7
Practice Exercise
# Questions: 10 Duration: 20 mints Negative Marking: 0.25 Calculator: Not Allowed

1. Two students appeared at an examination. One of them secured 9 marks more than the other and
his marks was 56% of the sum of their marks. The marks obtained by them are:
A. 39, 30 B. 41, 32 C. 42, 33 D. 43, 34
2. A fruit seller had some apples. He sells 40% apples and still has 420 apples. Originally, he had:
A. 588 apples B. 600 apples C. 672 apples D. 700 apples
3. If 20% of a = b, then b% of 20 is the same as:
A. 4% of a B. 5% of a C. 20% of a D. None of these
4. In a certain school, 20% of students are below 8 years of age. The number of students above 8 years
of age is of the number of students of 8 years of age which is 48. What is the total number of
students in the school?
A. 72 B. 80 C. 120 D. 100
5. Two numbers A and B are such that the sum of 5% of A and 4% of B is two-third of the sum of 6% of
A and 8% of B. Find the ratio of A : B.
A. 2 : 3 B. 1 : 1 C. 3 : 4 D. 4 : 3
6. In an election between two candidates, one got 55% of the total valid votes, 20% of the votes were
invalid. If the total number of votes was 7500, the number of valid votes that the other candidate
got, was:
A. 2700 B. 2900 C. 3000 D. 3100
7. Three candidates contested an election and received 1136, 7636 and 11628 votes respectively. What
percentage of the total votes did the winning candidate get?
A. 57% B. 60% C. 65% D. 90%
8. Two tailors X and Y are paid a total of Rs. 550 per week by their employer. If X is paid 120 percent of
the sum paid to Y, how much is Y paid per week?
A. Rs. 200 B. Rs. 250 C. Rs. 300 D. None of these
9. Gauri went to the stationers and bought things worth Rs. 25, out of which 30 paise went on sales tax
on taxable purchases. If the tax rate was 6%, then what was the cost of the tax free items?
A. Rs. 15 B. Rs. 15.70 C. Rs. 19.70 D. Rs. 20
10. Rajeev buys good worth Rs. 6650. He gets a rebate of 6% on it. After getting the rebate, he pays sales
tax @ 10%. Find the amount he will have to pay for the goods.
A. Rs. 6876.10 B. Rs. 6999.20 C. Rs. 6654 D. Rs. 7000

Check if you got them RIGHT !!! 

1. C 2.D 3.A 4.D 5.D 6.A 7.A 8.B 9.C 10.A

8
PROFIT, LOSS & DISCOUNT

Let us first understand the basic concepts!!!

The basis of commerce is the buying and selling of things. A person buys things and would like to sell them and
thereby make a profit on them. The price at which he buys is called the cost price (CP) and the price at which he sells
is called the selling price (SP).
The difference between these is profit. So, if one buys a watch for Rs 25 and sells for Rs 30, the profit made is (30-25)
= 5.
We can thus say that Profit = SP - CP.

Cost Price (CP): The price at which an article is bought, including all costs such as transportation, taxes, etc.
Selling Price (SP): The price at which an article is sold.
Profit or Gain: If SP is greater than the CP, the seller makes a profit or gain. Gain = (SP) - (CP)
Loss: If SP is less than the cost price, the seller makes a loss. Loss = (CP) - (SP)

Basic Formulae:
(i) Gain = (SP) - (CP)
(ii) (ii) Loss = (CP) - (SP)
(iii) (iii) Gain % = [(SP - CP) × 100]/CP).
(iv) (iv) Loss % = [(CP - SP) × 100]/CP)
Note:
1. If an article is sold at a gain of 20%, then SP = (120% of CP), or 1.2 × CP, or 6/5 × CP.
2. If an article is sold at a loss of 20%, then SP = (80% of CP), or 0.8 × CP, or 4/5 × CP.

Attention Please!!!
Note that the ability to move from fractions to decimals is of prime importance. Sums can be done quickly by
using one or the other.

Concept 1: Profit or loss is always calculated on the Cost Price


This example illustrates the mistake if profit is calculated on SP instead of CP

Illustration 1:
A person claims to have earned a profit of 10%. But it was found that he had calculated this profit per cent on SP
instead of CP. What is his actual profit?

Solution:
Let SP = 100, Profit = 10, hence CP = SP - Profit = 90.
Actual profit = 10/90 = 11 1/9%.
Note that if the base is changed, we will get a different answer, hence it is important to always calculate profit on CP.
9
Concept 2: To find SP when CP and Profit/Loss is given
Illustration 2:
If I buy an article for Rs 300 what will its SP be if I make a (a) 10% profit, (b) 20% loss?
Solution:
(a) CP = Rs 300; Profit = 10% of 300 = 30. Hence SP = 300 + 30 = 330

(b) CP = Rs 300; Loss = 20% of 300 = 60. Hence SP = 300 - 60 = 240.

The same calculation can be done as follows: CP (1 + Profit %) = 300 (1 + 10%) = 300 (1.10) = 330.

In the second case, it becomes 300 (1 - 20%) = 300 (0.8) = 240.

This calculation
HU eliminates one step so it is advisable to learn this.

Concept 3: To find CP, when SP and Profit/Loss are given


Illustration 3:
An article is sold for Rs 600. Find CP if it is sold for (a) 20% profit, and (b) 20% loss.

Solution:
(a) SP = Rs 600; Let CP = 100. Then with 20% profit SP = 120.

Using unitary method, we get: If SP is 120, CP = 100 If SP is 600, CP= ?


A quick calculation gives us that CP = 100/120 × 600 = 500.

(b) Let CP = 100. Then with 20% loss, we get SP = 80. If SP is 80, CP = 100. If SP is 600, CP = ? Again by using unitary
method, we get CP = 100/80 × 600 = 750.
The above can also be done as follows: (a) 20% profit means CP + 20% of CP = 1.2 CP = 600,
hence CP = 600/1.2 = 500.
In case of (b) we get CP - 20%(CP) = 0.8 CP = 600, hence CP = 600/0.8 = 750.

Illustration 4:
The cost price of 20 articles is the same as the selling price of x articles. If the profit is 25%, then the value of x is:

Solution:
Let S.P. of x articles = Rs. 100= C.P. of 20 articles
C.P. of one article = Rs. 5
Profit = 25%
S.P. of one article = 6.25
S.P. of x articles = 6.25x
6.25x = 100 => x = 100/6.25 = 16.

10
Concept 4 - Marked Price
When we go to a shop, we look at the goods which are labeled with price tags. This is the Marked Price (MP) of
the articles. However, this is not the Selling Price. The shopkeeper may give us a discount on the labeled price so
the Selling Price is arrived at after deducting Discount on the Marked Price.

Illustration 5:
In one shop, an article is marked 75% above the cost price, but the purchaser is allowed a discount of 20% on the
marked price. In another shop, a similar article is sold for Rs. 58 at a gain of 45%. What did the purchaser pay for
this article in the first shop?

Solution:
In the 1st shop:
Let C.P. = Rs. X

In the 2nd shop: S.P. = Rs. 58, Gain = 45%


∴ C.P.= (58*100)/(100+45) = 40
∴ S.P. in the 1st shop = 40 + 40% of 40 = Rs. 56.

Concept 5 - Selling Two Things at Same SP and Same Gain/Loss%


If two items are sold each at Rs X, one at a gain of p% and the other at a loss of p%, then the two transactions
have resulted in an overall loss of p2 /100 %.
Illustration 6:
A man sells two radios for Rs 924 each. On one he gains 12% and on the other he loses 12%. How much does he
gain or lose on the whole?
Solution:
Using the formula given above we can get the man’s loss, which is (122/100) = 1.44%.
NOTE – Selling Price is IMMATERIAL in such questions!!
If we do the sum by finding SP of the two articles, it becomes lengthy.

11
Concept 6 - When Quantity is Reduced
Illustration 7:
A grocer bought 10 Kg of apples for Rs. 81 out of which one Kg were found rotten. If he wishes to make a profit of
10%, then he should sell it at what price?
Solution:
CP = Rs 81. He wants to make 10% profit. Sales price after profit of 10% = 81 + 10% (81) = 89.10. But 1 kg is
rotten, so he has to sell 9 kg. Then sales price per kg = 89.10/9 = Rs 9.90 per kg.

Concept 7 - Trade Discount


To attract customers it is a common practice to announce discount on the marked price of an article. The
discount is always taken as a % of the marked price only unless otherwise specified.
Illustration 8:
Suppose the list price of an article be Rs 450. A discount of 5% on its list price is announced.
Solution:
Hence the new selling price = (95*450)/100 = Rs 422.5

Concept 8 - Cash Discount


In addition to trade discount, the manufacturer may offer an additional discount called the Cash Discount if the
buyer makes full payment within a certain specified time.
Cash Discount is usually offered on the net price (the price after subtracting discount from the marked price).
Therefore, Cash Price = Net Price - Cash Discount
Cash discount is always calculated on net price, unless otherwise specified.
Illustration 9:
A dealer offers a cash discount of 20% and still makes a profit of 20%, when he further allows 16 articles to a
dozen to a particularly sticky bargainer. How much percent above the cost price were his wares listed?
Solution
Let the MP be 100, then SP for 16 articles is 80, after a 20% discount.
SP for 12 articles is 80 × 12/16 = 60. This includes a profit of 20%, hence the CP must be 60/1.2 = 50. Marked price
is 100, which is (100 – 50)/50 × 100 = 100% over the CP.

12
Concept 9 - Wrong Weight
When a tradesman professes to sell at cost price, but uses a false weight,
then the percentage profit earned = (100 × error)/(true weight – error)

Illustration 10:
A dishonest dealer professes to sell his goods at cost price, but uses a weight of 960 gm for a kg weight. Find his
gain per cent.

Solution:

To explain, let us say that he has 1000 gm to sell and the CP is Rs 1 per gm. When a customer asks for 1 kg, he
charges CP, which is Rs 1000 but gives only 960 g. In this case, SP = 1000 and CP is only Rs 960. So he makes a
profit of Rs 1000 – 960 = Rs 40.

Profit percentage = 40/960 × 100 = 4 1/6%.

We can also use the above formula to arrive at the same answer.

Concept 10 - Successive Discounts


When a tradesman offers more than one discount to the customer, then sometimes you need to calculate the
single discount, which is equal to the two discounts given.

METHOD 1: Start with 100 and keep applying the discounts to the decreased price.
METHOD 2 : Single discount, which is equal to two successive discounts of M% and N% = [M + N – (MN/100)] %.

Illustration 11:
A tradesman offers two successive discounts of 20% and 10%, which single discount is equal to these two
successive discounts?

Solution:
Let the marked price is Rs. 100.
After the first discount of 20% the remaining price is 80 and after the second discount of 10%, the remaining part
is 80 – 8 = 72. Hence single discount is 100 – 72 = 28 i.e. 28%.

SHORT - CUT

Single discount, which is equal to two successive discounts of 20% and 10% = [20 + 10 – (200/100)] %.
= (30-2) % i.e. 28%

13
Practice Exercise
# Questions: 10 Duration: 20 mints Negative Marking: 0.25 Calculator: Not Allowed

1. A man sold two houses for Rs 3,75,890 each. On one he gains 15% and on the other he loses 15%.
How much does he gain or lose in the whole transaction?
A. 2.25% loss B. 2.50% gain C. 1.15% loss 2.25% gain
Check if you Check if you got them RIGHT !!! 
2. Alfred buys an old scooter for Rs. 4700 and spends Rs. 800 on its repairs. If he sells the scooter for Rs.
5800, what is his gain percent?
got them
A. 4 4/7% B. 5 5/11% C. 10% D.12%
1. A 2. B 3. B 4. B 5. C
3. 6.TheAcost price of 20
7. C articles is the same
8. D as the selling
9. B price of x articles.
10. C If the profit is 25%, then what
is the
11. A value of x?
12. B 13. B 14. B 15. C
A. 15 B. 16 C. 18 D. 25

4. In a certain store, the profit is 320% of the cost. If the cost increases by 25% but the selling price
remains constant, approximately what percentage of the selling price is the profit?
A. 30% B. 70% C. 100% D. 250%

5. A vendor bought toffees at 6 for a rupee. How many for a rupee must he sell to gain 20%?
A. 3 B. 4 C. 5 D. 6

6. The percentage profit earned by selling an article for Rs. 1920 is equal to the percentage loss
incurred by selling the same article for Rs. 1280. At what price should the article be sold to make 25%
profit?
A. Rs. 2000 B. Rs. 2200 C. Rs. 2400 D. Data inadequate

7. A man buys a cycle for Rs. 1400 and sells it at a loss of 15%. What is the selling price of the cycle?
A. Rs. 1090 B. Rs. 1160 C. Rs. 1190 D. Rs. 1202

8. On selling 17 balls at Rs. 720, there is a loss equal to the cost price of 5 balls. What is the cost price
of a ball?
A. Rs. 45 B. Rs. 50 C. Rs. 55 D. Rs. 60

9. A trader mixes 26 kg of rice at Rs. 20 per kg with 30 kg of rice of other variety at Rs. 36 per kg and
sells the mixture at Rs. 30 per kg. His profit percent is:
A. No profit, no loss B. 5% C. 8% D. 10%

10. When a plot is sold for Rs. 18,700, the owner loses 15%. At what price must that plot be sold in
order to gain 15%?
A. Rs. 21,000 B. Rs. 22,500 C. Rs. 25,300 D. Rs. 25,800

1. A 2. B 3. B 4. B 5. C
6. A 7. C 8. D 9. B 10. C

14
Review Exercise 1
Simple & Compound Interest
1.# Questions:
a 5 Duration:
2. c 10 mints 3. dNegative Marking:
4. 0.25
b Calculator: Not
5. Allowed
d
Q 1. 1.
Rs.25,000
a is invested 2.
for 3c years at 12% compound
3. d interest p.a. 4.
What
b is the interest in5.thedthird year?
A. Rs.3,763 B. Rs.3,110 C. Rs.3,000 D. Rs.3,450

Q2. If the difference between CI and SI for two years is Rs. 360 at 12% then find the Principal.
A. Rs.12,000 B. Rs.13,110 C. Rs.10,000 D. Rs.14,450

Q3. Sandeep borrowed Rs. 12,000 at 10% p.a. He repaid Rs. 7000 at the end of the 1st year. What amount
should he pay at the end of the 2nd year to completely discharge the loan, CI is being reckoned?
A. Rs.3,720 B. Rs.6,720 C. Rs.8,260 D. Rs.6,820

Q4. Kashi borrowed Rs. 12,000 at 10%p.a. rate of Compound Interest. He repaid a certain amount at the end
of the first year. Then he paid Rs. 10,120 at the end of the 2nd year to completely discharge the loan. What
amount did he repay at the end of first year?
A. Rs.3,000 B. Rs.4,000 C. Rs.5,000 D. Rs.2,050

Q5. Pradeep borrowed Rs 12,600 at 10% rate of compound interest. If this amount has to be repaid in two
equal annual installments, find the value of each installment.
A. Rs.7,763 B. Rs.7,210 C. Rs.3,000 D. Rs.7,260

1. a 2. c 3. d 4. b 5. d

15
1.B 2.D 3.A 4.C Review
5.C Exercise
6.C 2
7.B 8.D 9.C 10.B

Number Series
# Questions: 10 Duration: 10 mints Negative Marking: 0.25 Calculator: Not Allowed

Find odd man out in given series (Q1-Q5):-

1) 121, 102, 84, 72, 61, 54


A. 102 B. 84 C. 72 D. all are right
2) 5, 6, 14, 45, 184, 925
A. 45 B. 925 C. 184 D. all are right
3) 764, 620, 520, 456, 400, 404
A. 400 B. 404 C. 456 D. all are right
4) 1337, 662, 331, 164, 81, 40
A. 164 B. 81 C. 662 D. all are right.
5) 3, 5, 11, 14, 17, 21
A. 21 B. 17 C. 14 D. 3

Find the next two terms of the given series (Q6-Q10):-

6) 42, 40, 38, 35, 33, 31, 28, ?, ?


A. 25, 22 B. 26, 23 C. 26, 24 D. 25, 23
7) 8, 12, 9, 13, 10, 14, 11, ?, ?
A. 14, 11 B. 15, 12 C. 8, 15 D. 15, 19
8) 36, 31, 29, 24, 22, 17, 15, ?, ?
A. 13, 11 B. 10, 5 C. 13, 8 D. 10, 8
9) 3, 5, 35, 10, 12, 35, 17, ?, ?
A. 22, 35 B. 35, 19 C. 19, 35 D. 19, 24
10) 13, 29, 15, 26, 17, 23, 19, ?, ?
A. 21, 23 B. 20, 21 C. 20, 17 D. 25, 27

1.B 2.D 3.A 4.C 5.C 6.C 7.B 8.D 9.C 10.B

16
Review Exercise 3
Letter Series
# Questions: 10 Duration: 10 mints Negative Marking: 0.25 Calculator: Not Allowed

In alphabet series, some alphabets are missing which are given in that order as one of the alternatives below it.
Choose the correct alternative (Q1-Q5):-

1) _ _ aba _ _ ba _ ab
A. abbba B. abbab C. baabb D. bbaba
2) aa _ ab _ _ aaa _ a
A. aaab B. aabb C. abab D. baaa
3) a _ bbc _ aab _ cca _ bbcc
A. bacb B. acba C. abba D. caba
4) mnonopqopqrs _ _ _ _ _
A. mnopq B. oqrst C. pqrst D. qrstu
5) _ A D A C B _ _ B D C C 1 3 _ _ 1 2 4 2 _ _ _ _ a _ _ b _ _ c d
A. a, c, d, d B. a, d, c, c C. c, a, d, d D. d, c, a, a

In these series, you will be looking at both the letter pattern and the number pattern. Fill the blank in the middle of
the series or end of the series (Q6-Q10):-

6) CMM, EOO, GQQ, _____, KUU


A. GRR B. GSS C. ISS D. ITT
7) QPO, NML, KJI, _____, EDC
A. HGF B. CAB C. JKL D. GHI
8) P5QR, P4QS, P3QT, _____, P1QV
A. PQW B. PQV2 C. P2QU D. PQ3U
9) QAR, RAS, SAT, TAU, _____
A. UAV B. UAT C. TAS D. TAT
10) ELFA, GLHA, ILJA, _____, MLNA
A. OLPA B. KLMA C. LLMA D. KLLA

1.B 2.A 3.B 4.C 5.D 6.C 7.A 8.C 9.A 10.D

17
RATIO & PROPORTION

Let us first understand the basic concepts!!!

RATIO
The ratio of two quantities in the same units is a fraction that one quantity is of the other. Thus, a to b is a ratio
(a/b), written as a : b.

The first term of a ratio is called antecedent, while the second term is known as consequent. Thus, the ratio 4:7
represents 4/7 with antecedent 4 and consequent 7.

The multiplication or division of each term of a ratio by a same non-zero number does not affect the ratio.
Thus, 3 : 5 is the same as 6 : 10 or 9 : 15 or 12 : 20 etc.

PROPORTION
The equality of two ratios is called proportion.
Thus, 2 : 3 = 8 : 12 is written as 2 : 3 : : 8 : 12 and we say that 2, 3, 8 and 12 are in proportion.
In a proportion, the first and fourth terms are known as extremes, while second and third terms are known as
means.
In a proportion, we always have:
Product of Means = Product of Extremes.

Concept 1: Fourth Proportional


Fourth proportional means: the fourth number which has the same ratio with the third number as the second
number has with the first number.

Illustration 1:
Find the fourth proportional to 8, 12 and 16.

Solution
You are supposed to calculate a number, which is having the same ratio with 16 as 12 is having with 8 i.e.

In this case, the product of extremes, would be equal to product of middle values. 8×x=12×16⇒x =24

18
Concept 2: Second, or Mean Proportional
Second proportional means: the second number which has the same ratio with the first number as the third
number has with it.
This is also known as mean proportional.
If a:b = b:c, then b is called the mean proportional and is equal to the square root of the product of a and c.

Illustration 2:
Find the mean proportional between 4 and 16.
Solution
Mean proportional between 4 & 16 = √4×16 = √64 = 8

Concept 3: Dividing Numbers Into Ratios


If any number/amount is to be divided in the ratio of a : b.
Then their respective shares can be calculated as ⇒ number × a/(a+b) and number × b/(a+b).
Illustration 3:
Divide Rs 455 in the ratio 4 : 3.
Solution:
To divide a sum in any ratio, we divide by the sum of the ratio and multiply by the parts individually.
Hence one part = 455 × 4/7 = 260
Second part = 455 × 3/7 = 195.

Illustration 4:
A certain amount was divided between Kavita and Sheena in the ratio 4 : 3. If Sheena’s share was Rs 2400, the
amount is?
Solution
We can make a simple equation: Sheena’s share is 3/7 x = 2400, then x = 2400 × 7/3 = 5600.

Concept 4: Resolving Ratios with 3 Numbers

When three terms are given, we can resolve them by finding the LCM of the common term.

Illustration 5:
If a : b = 2 : 3 and b : c = 5 : 7, find a : c and a : b : c.

Solution:
The LCM of the common term b (3, 5) is 15.
Then a : b = 10 : 15 and b : c = 15 : 21.
Hence we can resolve the ratio as: a : b : c = 10 : 15 : 21.
Similarly, we can resolve more than three ratios also.

19
Concept 5: Using Proportion in Sums

When any ratio is given, we can assume the terms as multiples of ratio.
Illustration 6: Find three numbers in the ratio 2 : 3 : 5, the sum of whose squares is 608.
Solution:
Take the terms as 2x: 3x:5x. Then sum of the squares = 4x +9x +25x = 608. On solving we get 38x = 608 and x = 4.

Illustration 7: If 5m-n = m+2n, find the value of (4m+n) : (4m-n).


Solution: 5m – n = m + 2n, hence 4m = 3n. Substitute in the given expression to get: Required ratio = (3n + n : 3n
– n) = 4n : 2n, hence the ratio is 2 : 1.

Concept 6: Sums Using Mixtures and Liquids


To solve sums on liquids, make a fraction and add the liquid to the relevant term.
Illustration 8:
A mixture contains alcohol and water in the ratio 4 : 3. If 7 litres of water is added to the mixture, the ratio of
alcohol and water becomes 3 : 4. Find the quantity of alcohol in the mixture.
Solution: Let the alcohol : water be 4x : 3x.
Adding 7 litres of water, the fraction becomes 4x/(3x + 7) = 3⁄4. On solving, we get x = 3 and alcohol = 4x = 12.

Concept 7: Variation

If one object ‘m’ is directly related with the other object ‘n’, then this is written as m = kn, where ‘k’ is a
constant. Similarly if the object is inversely related with the other object, then m = k/n.A ∝ B (A varies as B)
Then, A = kB, where k is a constant.

A quantity A is said to vary inversely as quantity B when A varies directly as the reciprocal of B.
A = k/B where k is constant A quantity is said to vary jointly as number of others when it varies directly as their
product. A = k(B × C). A is said to vary directly as B and inversely as C if A varies as B/C.
Direct Variation: If two variables a and b are related in such a way that for any increase or decrease in “a”, “b”
will also increase or decrease and vice versa, then the two magnitudes are in direct variation or direct
proportion to each other.
Inverse Variation: If two variables “a” and “b” are related in such a way that for any increase (or decrease) in
“a”, quantity “b” decreases (or increases) in the same ratio, then these two magnitudes are said to vary
inversely to each other and the proportion in that case is called inverse proportion or inverse variation

Illustration 9: The marks of a person are directly related with the number of hours a person spends in the
studies everyday. Ramesh scored 80%, after studying 4 hrs a day. What would be the marks of Sudesh, who
spends 3 hrs a day on studies?
Solution: Let the marks scored by a person be = m, and the hrs he spends = h.
As marks are directly related with hrs, we can write m = kh. When ‘m’=80, then‘h’is4 ⇒80=4k.Sovalueofk =20.
Now for Sudesh, when h is given to be 3 then m = 20h ⇒ 20×3 = 60%
Illustration 10: A = 4 when B = 24, find B when A = 7, where B varies directly as A.
Solution: Since B varies directly as A, we can write B = kA where k is a constant. Hence 24 = k × 4, on solving we
get k = 4. The equation is thus: B = 4A. If A = 7, B = 28.

20
Practice Exercise
# Questions: 10 Duration: 20 mints Negative Marking: 0.25 Calculator: Not Allowed

1. A stick 1.4 m long casts a shadow 1.3 m long and the same time when a pole casts a shadow 5.2 m
long. Find the length of the pole.
A. 5.6 m B. 6.5 m C. 7.5m D. 5.0m
2. A girl got 257 marks in English, Mathematics and History together. The ratio of the marks obtained
by her in English and Mathematics is 7 : 8, while that in English and History is 12 : 11. Find the
number of marks obtained by her in each subject.
A. 75 B. 77 C. 78 D. 85
3. The ratio of the first and second-class fares between the two stations is 4 : 1 and the number of
passengers traveling by first and second-class is 1 : 40. If Rs. 1100 is collected as fare, what is the
amount collected from first class passengers?
A. 102 B. 101 C. 100 D. 110
4. 729 ml of a mixture contains milk and water in the ratio 7 : 2. How much more water is to be added
to get a new mixture containing milk and water in the ratio of 7 : 3?
A. 60 ml B. 70 ml C. 81 ml D. 90 ml
5. Square of x varies as cube of y. Given that x = 4 when y = 2, find x when y is 4.
A. 16 B. 9 C. 25 D. 36
6. Area of a triangle varies as its base when height is constant and as height when base is constant. If
the base is 4 cm and height is 10 cm, the area of the triangle is 20 cm2. Find the length of the base if
the area of the triangle is 25 cm2 and its height is 5 cm.
A. 20 B. 50 C. 10 D. None of these
7. If y is proportional to x and x = 6 when y = 2; Find The value of y when x = 15.
A. 15 B. 10 C. 5 D. 3
8. A and B together have Rs. 1210. If of A's amount is equal to of B's amount, how much amount
does B have?
A. Rs. 460 B. Rs. 484 C. Rs. 550 D. Rs. 664
9. Two numbers are respectively 20% and 50% more than a third number. The ratio of the two numbers
is:
A. 2 : 5 B. 3 : 5 C. 4 : 5 D. 6 : 7
10. Seats for Mathematics, Physics and Biology in a school are in the ratio 5 : 7 : 8. There is a proposal to
increase these seats by 40%, 50% and 75% respectively. What will be the ratio of increased seats?
A. 2 : 3 : 4 B. 6 : 7 : 8 C. 6 : 8 : 9 D. None of these

1. A 2. B 3. C 4. C 5. A
6. C 7. C 8. B 9. C 10. A

21
ALLIGATIONS

Let us first understand the concept of Alligations!!!

The technique of alligation is applicable in all the cases where two extreme values are given and one average value
is given. It is a very useful technique which can be applied in chapters like Percentage, Simple interest, Ratio &
proportion, Average etc.
This technique enables us to calculate the ratio in which extreme values/ prices/ interests/ ratios and averages
should be mixed so that a given average value/price/interest/ratio and average can be obtained.

Allegation is the rule that enables us to find the proportion in which the two or more ingredients at the given price
must be mixed to produce a mixture at a given price. Thus,

Find it complicated to remember the Formula!! Don’t worry, keep in mind the below short cut by following the
direction of the arrows:

Attention please!!
1. Mean price is always less then dearer price and is always more than cheaper price.
2. The price of the first kind should always be on the left hand side.

22
Time to solve some Questions !!!
Concept 1: Questions Based on the Rule of Alligation
Illustration 1:
In what proportion must a grocer mix tea at Rs 125 per kg with tea at Rs 150 per kg, so that the mixture is worth
Rs 130 a kg?

Solution:
CP of 1 Kg (C) CP of 1 Kg (D)
125 150

Mean Price
130

(150-130)=20 (130-125)= 5

Ignore the Minus sign.


Follow the direction of the arrows and subtract the mean price from the two CPs. Write the subtracted figure at
the bottom. The bottom figure represents the ratio of the quantities that the two must be mixed. In this case the
ratio of cheaper to dearer is 20 : 5. They must be mixed in the ratio 4:1.

Illustration 2:
In what proportion should tea worth Rs. 20 be mixed with tea worth Rs. 14, so that the average price per kg of tea
is Rs. 16 per kg?
Solution:
The two extreme prices are Rs.14 and Rs. 20 and average price is Rs.16. Put the extreme values on two ends and
average in the centre. Then subtract in the direction of the arrows and arrive at the resultant values (given at the
bottom of the cross):

Ratio ⇒ 4 : 2 = 2 : 1
Thus, this case the ratio of cheaper to dearer is 2:1. However, in the question, we need to find out the ratio of tea
worth Rs. 20 to tea worth Rs. 14 i.e. dearer to cheaper. Thus in the above Illustration, answer is 2 : 1 and not 1 : 2.

Attention Please!!!
Keep in mind the simple point that the order of the ratio follows the order of what is written at the
top.

23
Concept 2: To find the value based on the Ratio given
Illustration 3:
In an organization the average salary of employees is Rs 800. The average salary of 10 officers is Rs 3000 and the
average salary of clerks is Rs 600. What is the number of clerks in the organization?

Solution: 600 3000


Mean Salary
800
2200 200

11 : 1

From the above, the ratio is 11:1 means for 1 officer there are 11 clerks in the organization.
So for 10 officers there would be 11 × 10 /1 = 110 clerks in the organization.

Concept 3: Ratio in which ingredients of two vessels be mixed to get the desired
ratio
Illustration 4:
The ratio of water and alcohol in two different containers is 2:3 and 4:5. In what ratio we are required to mix the
mixtures of two containers in order to get the new mixture in which the ratio of alcohol and water be 7:5?

Solution:
The ratio of water to total in the three cases is 2/5, 4/9 and 7/12.

Find the LCM of the denominators (2, 9 and 12) i.e. 180 to equate the denominators and simplify the fractions
to get 72/180, 80/180 and 75/180.
Applying the rule of alligations:

5 : 3
Therefore, the ratio is 5: 3
24
Concept 4: Finding the quantity to be added to gain x% profit
Illustration 5:
A shopkeeper has 100 kg of salt. He sells part of the total quantity A at 7% profit and the rest B at 17 % profit. If
he gains 10 % profit on the whole quantity, then find how much is sold at 7 % profit?
Solution:
Assume that A and B are two parts of the mixture. To determine the quantity A and B, first calculate ratio of A :
B.
Given:
1) Selling price of mixture with 10% profit = Rs. 110
2) With 17 % profit, the selling price of A = Rs. 117
3) With 7 % profit, the selling price of B = Rs. 107

Now, this question can be easily solved by using the rule of alligation. As, the ratio of A : B = 3 : 7
Let the quantity of part A be 3x and part B be 7x in the total quantity of 100 kg.
Therefore, 3x + 7x = 100
10x = 100 => x = 10
Quantity of part A = 3x = 3 x 10 = 30 kg
Quantity of part B = 7x = 7 x 10 = 70 kg

Practice Exercise
1. In what ratio, water must be mixed with fruit juice costing Rs.24 per litre so that the juice would be worth of
Rs.20 per litre?
A. 1:4 B.1:5 C.1:6 D.2:5
2. Box A contains wheat worth Rs.30 per kg and box B contains wheat worth Rs.40 per kg. If both A and B are
mixed in the ratio 4:7 then what is the price of mixture per kg ?
A. Rs. 36.36 B.Rs. 35.80 C.Rs. 42.50 D.Rs.31.30
3. A seller has two variety of rice worth Rs.80 per kg and Rs.95 per kg. If he gets 20% profit by selling the
mixture of two varieties at Rs.84 then what is the mixing ratio ?
A. 7:5 B. 5:3 C. 7:2 D.5:2
4. There are two varieties of tea worth Rs.90 per kg and Rs.70 per kg. If X kg of 1st kind is mixed with 56 kg of
2nd kind to get a profit 25% by selling the mixture at Rs.95 per kg then X equals to ?
A. 35 B. 24 C. 48 D.19

1. B 2. A 3. D 4. B

25
MIXTURES

What are Mixtures??


Mixture or alloys contains two or more ingredients of certain quantity mixed together to get a desired
quantity. The quantity can be expressed as a ratio or percentage. For ex: 1 liter of a mixture contains 250ml
water and 750 ml milk. That means, ¼ of mixture is water and ¾ of mixture is milk. In other words, 25% of
mixture is water and 75% of mixture is milk.

Concept 1: Finding the quantity of an ingredient in the mixture


Illustration 1:
A mixture contains alcohol and water in the ratio 4 : 3. If 7 litres of water is added to the mixture, the ratio of
alcohol and water becomes 3 : 4. Find the quantity of alcohol in the mixture.

Solution:
Let the alcohol : water be 4x : 3x.
Adding 7 litres of water, the fraction becomes 4x/(3x + 7) = 3⁄4. On solving, we get x = 3 and alcohol = 4x = 12.

Concept 2: Quantity of ingredient to be added to increase the content of


ingredient in the mixture to y%
Illustration 2:
A mixture of water and milk contains 80% milk. In 50 litres of such a mixture, how many litres of water is
required to increase the percentage of water to 50%?

Solution:
Total mixture = 50 litres Milk = 80% of 50 = 40 litres Water = 20% of 50 = 10 litres
Let ‘x’ litres of water is added.
Now, milk = 40 litres
Water = 10+x
Total = 50+x
Now, 50% of total = Water
½ x (50 + x) = 10 + x
x = 30 litres

26
Concept 3: Quantity of ingredient to be added to change the ratio of ingredients
in a mixture
Illustration 3:
729 ml of a mixture contains milk and water in the ratio 7 : 2. How much more water is to be added to get a new
mixture containing milk and water in the ratio of 7 : 3?
Solution:
Milk and water in the original liquid = 7/9 × 729 = 567 and water = 2/9 × 729 = 162.
Let water to be added = x.
Then, 567/(162 + x) = 7/3
Hence, we get 1701 = 1134 + 7x; or 7x = 567; or x = 81

Concept 4: Replacement of a part of a solution


If a vessel contains A liters of milk and if B litres of milk is withdrawn and replaced by water, and again if B litres
of mixture is withdrawn and replaced by water and this operation is replaced n times in all, then

(Quantity of milk left after nth operation) (A – B) n

=
(Initial quantity of milk) A

Thus, quantity of milk/alcohol left after nth operation = [A(1 – (B/A))n]


Or in other words,

Final Amount of ingredient that is not replaced =

27
Let’s understand the concept 4 with an Illustration
Illustration 4:
10 gallons are drawn from a container full of alcohol and filled with water again. 10 gallons of mixture are again
drawn and the container is filled with water again. If the ratio of alcohol and water left in the container is
49 : 32, then find how much quantity does the container hold?

Solution:
Initially the container contains only wine. 10 gallons of alcohol was removed and same quantity of water was
added. This process is again repeated by replacing the mixture( alcohol + water) of 10 gallons with same
quantity of water. Hence, the initial quantity of wine and the final quantity of water and alcohol is the same.

Follow the below mentioned steps:

Therefore,
n
(Quantity of alcohol left after nth operation) (A – B)
=
(Initial quantity of alcohol) A
49/81 = (A-10)2/A

Solving, we can find the value of A (initial quantity of alcohol)


A = 45 gallons

28
Practice Exercise
# Questions: 5 Duration: 10 mints Negative Marking: 0.25 Calculator: Not Allowed

1. A vessel is filled with liquid, 3 parts of which are water and 5 parts syrup. How much of the mixture must
be drawn off and replaced with water so that the mixture may be half water and half syrup?
A. 1/3 B.1/4 C.1/5 D.1/7

2. A can contains a mixture of two liquids A and B is the ratio 7 : 5. When 9 litres of mixture are drawn off
and the can is filled with B, the ratio of A and B becomes 7 : 9. How many litres of liquid A was contained by
the can initially?
A. 10 B.21 C.20 D.25

3. How many litres of water should be added to a 30 litre mixture of milk and water containing milk and
water in the ratio of 7 : 3 such that the resultant mixture has 40% water in it?
A. 5 Lts B. 10 Lts C. 7 Lts D.None of these

4. In a mixture of 75 litres, the ratio of milk to water is 2 : 1. What is the amount of water to be further added
to the mixture so as to make the ratio of the milk to water 1 : 2 ?
A. 45 Lts B. 60 Lts C. 75 Lts D.80 Lts

5. A sample of x litres from a container having a 60 litre mixture of milk and water containing milk and water
in the ratio of 2 : 3 is replaced with pure milk so that the container will have milk and water in equal
proportions. What is the value of x?
A. 6 Lts B.30 Lts C. 20 Lts D. 10 Lts

1. C 2. B 3. A 4. C 5. D

29
PARTNERSHIP
What is Partner
When two or more than two persons run a business jointly, they are called partners and the deal is known as
partnership.
Attention Please!!!! Partnership is closely related to Ratios. You must be thorough in your understanding
of ratios before doing sums on partnership.

Concept 1: Distributing Profits when invested for the same time period
If the capitals of the partners remain in the business for the same duration of time then the profit is simply
distributed in the ratio of their capitals.

Illustration 1:
Three partners A, B, C invest Rs. 26000, Rs. 34000 and Rs. 10000 respectively in a business.
Out of a profit of Rs. 3500, what is B's share?

Solution:
B’s share will be = (B’s Investment /Total Investment) × Total profit = (34000/70000) × 3500 = Rs.1700.

Concept 2: When Partners Join at Different Times


If the different partners join at different points of time then their profit sharing ratio is calculated by multiplying
the capital invested with the number of months the capital remains in the business.

Illustration 2:
Sanjay and Raj started a business and invested Rs. 20,000 and Rs. 25,000 respectively. After 4 months, Raj left
and Naresh joined by investing Rs. 15,000. At the end of the year, there was a profit of Rs 4,600. What is the
share of Naresh?

Solution:
Sanjay stays for 12 months, Raj stays for 4 months and Naresh joins after 4 months, so he stays for 8 months
Calculation of Profit sharing ratio : (Capital Invested) x (number of months the capital remains in the business)
Sanjay : 20,000 × 12
Raj : 25,000 x 4
Naresh :15,000 × 8
The ratio of sharing is thus 240 : 100 : 120.
Hence N’s share = 120/460 × 4600 = 1200.

30
Practice Exercise

Let’s try some questions without options!!


# Questions: 5 Duration: 10 mints Negative Marking: 0.25 Calculator: Not Allowed

1. Raj invested Rs 76000 in a business. After few months Monty joined him and invests Rs 57000. At the end
of year both of them share the profits at the ratio of 2:1. After how many months Monty joined Raj ?

2. A and B started a business by investing money in ratio of 5:6. C joined them after few months by sharing
an amount equal to B's share. At the end of year 20% profit was earned which was equal to Rs 98,000. How
much money was invested by C ?

3. A, B and C shared profits in ratio of 5:7:8. They partnered for 14 months, 8 months and 7 months
respectively. What was the ratio of their investments?

4. 4. A, B and C jointly thought of engaging themselves in a business venture. It was agreed that A would
invest Rs. 6500 for 6 months, B, Rs. 8400 for 5 months and C, Rs. 10,000 for 3 months. A wants to be the
working member for which, he was to receive 5% of the profits. The profit earned was Rs. 7400. Calculate
the share of B in the profit.

5. In a partnership, A invests 1/6 of the capital for 1/6 of the time, B invests 1/3 of the capital for 1/3 of the
time and C, the rest of the capital for the whole time. What is the share of B in the profit of Rs. 4600?

1. 4 months 2. 2,10,000 3.20:49:64 4. Rs. 2,660 5. Rs. 800

31
Blood Relations

Important Relations
Mother’s or father’s son = Brother
Mother’s or father’s daughter = Sister
Mother’s or father’s brother = Uncle
Mother’s or father’s sister = Aunt
Mother’s or father’s father = Grandfather
Mother’s or father’s mother = Grandmother
Son’s wife = Daughter-in-Law
Daughter’s husband = Son-in-Law
Husband’s or wife’s sister = Sister-in-Law
Husband’s or wife’s brother = Brother-in-Law
Brother’s son = Nephew
Brother’s daughter = Niece
Uncle or aunt’s son or daughter = Cousin
Sister’s husband = Brother-in-Law
Brother’s wife = Sister-in-Law
Grandson’s or Granddaughter’s daughter = Great grand daughter

Review Exercise 4
# Questions: 5 Duration: 10 mints Negative Marking: 0.25 Calculator: Not Allowed

1. A girl introduced a boy as the son of' the daughter of the father of her uncle. The boy is girl's
a) Brother b) Son c) uncle d) Son in law
2. In a family, there are six members A, B, C, D, E and F. A and B are a married couple, A being the male
member. D is the only son of C, who is the brother of A. E is the sister of D. B is the daughter-in-law of F,
whose husband has died. How is E related to C ?
a) Sister b) Daughter c) Cousin d) Mother
3. Q's mother is sister of P and daughter of M. S is daughter of P and sister of T. How is M related to T?
a) Father b) Grandfather c) Grandmother d) Either Grandmother or Grandfather
4. P x Q means P is the sister of Q ; P + Q means P is the father of Q ; P - Q means P is the mother of Q.
Which of the following means S is the aunt of T ?
a) T x M + S b) S + T x M c) S x M + T d) S x M + R – T
5. A + B means A is the daughter of B ; A x B means A is the son of B and A - B means A is the wife of B.
If P x Q - S, which of the following is true ?
a) S is wife of Q b) S is father of P c) P is daughter of Q d) Q is father of P

1. a 2. b 3. d 4. c 5. b

32
Review Exercise 5
Direction Sense Test
# Questions: 5 Duration: 10 mints Negative Marking: 0.25 Calculator: Not Allowed

1. Two cars start from the opposite places of a main road, 150 km apart. First car runs for 25 km and takes a right
turn and then runs 15 km. It then turns left and then runs for another 25 km and then takes the direction back to
reach the main road. In the mean time, due to minor break down the other car has run only 35 km along the main
road. What would be the distance between two cars at this point?
a) 65 km b) 75 km c) 80 km d) 85 km

2. A # B means B is at 1 metre to the right of A.


A $ B means B is at 1 metre to the North of A.
A * B means B is at 1 metre to the left of A.
A @ B means B is at 1 metre to the south of A.
In each question first person from the left is facing North.
According to M # N $ T, T is in which direction with respect to M?
a) North-West b) North-East c) South-West d) South-East

3. Rahul put his timepiece on the table in such a way that at 6 P.M. hour hand points to North. In which direction
the minute hand will point at 9.15 P.M. ?
a) South-East b) South c) North d) West

4. Amit started walking positioning his back towards the sun. After some time, he turned left, then turned right and
towards the left again. In which direction is he going now?
a) North or South b) North or West c) East or West d) South or West

5. A horse is facing north. It turns 90 degrees in the clockwise direction, then 180 degrees in the anti-clockwise and
then another 90 degrees in the same direction. Which direction is the horse facing now?
a) East b) South c) Southeast d) Southwest

1. a 2. b 3. d 4. a 5. b

33
Review Exercise 6
Coding Decoding
# Questions: 5 Duration: 5 mints Negative Marking: 0.25 Calculator: Not Allowed

1. If BOMBAY is written as MYMYMY, how will TAMIL NADU be written in that code?
a) TIATIATIA b) MNUMNUMNU c) IATIATIAT d) ALDALDALD

2. If ROSE is coded as 6821, CHAIR is coded as 73456 and PREACH is coded as 961473, what will be the code for
SEARCH?
a) 246173 b) 214673 c) 214763 d)216473

3. In a certain code, a number 13479 is written as AQFJL and 5268 is written as DMPN. How is 396824 written in
that code?
a) QLPNKJ b) QLPNDF c) QLPMNF d) QLPNMF

4. If orange is called butter, butter is called soap, soap is called ink, ink is called honey and honey is called orange,
which of the following is used for washing clothes ?
a) Honey b) Butter c) Orange d) Soap e) Ink

5. In a certain code language, 'col tip mot' means 'singing is appreciable' ; 'mot baj min' means 'dancing is good'
and 'tip nop baj' means 'singing and dancing' , which of the following means 'good' in that code language ?
a) not b) min c) baj d) Can’t be determined

1. b 2. b 3. d 4. e 5. b

34
Review Exercise 7
Average
# Questions: 5 Duration: 10 mints Negative Marking: 0.25 Calculator: Not Allowed

1. What is the average of x, 2x, and 6?


A : x /2 B : 2x C : (x+2)/ 6 D : x+2 E : (x+2)/ 3

2. If the average of five numbers is -10, and the sum of three of the numbers is 16, then what is the
average of the other two numbers?
A: –33 B: –1 C: 5 D: 20 E: 25

3. There are two sections A and B of a class, consisting of 36 and 44 students’ respectively. If the average
weight of section A is 40kg and that of section B is 35kg, find the average weight of the whole class.
A. 30 kg B. 35 kg C. 42.5 kg D. 37.25 kg

4. The average score of a cricketer for ten matches is 38.9 runs. If the average for the first six matches is
42, then find the average for the last four matches.
A. 33.25 B. 33.5 C. 34.25 D. 35

5. A Batsman makes a score of 87 runs in the 17th inning and thus increases his average by 3. Find his
average after 17th inning.
A. 40 B. 39 C. 52 D. 55

1. d 2. a 3. d 4. c 5. b

35
Time and Work

Formula required for Time and Work problems:


1.If ‘A’ can do the work in ‘n’ days , Then A’s one day work is 1/n.
2. If A’s 1 day’s work is 1/n, then A can finish the work in ‘n’ days.

Example Problems on Time and Work with solutions:


Example 1:
If A can finish a work in 2 days, B can finish the work in 3 days, then How many days together takes to
finish the work?
1. General Method:
Answer: A’s one day work is ½ & B’s one day work is 1/3
Now A and B work together in one day is (1/2)+(1/3) = 5/6
In one day A and B together work in one days is 5/6, So number of days they take together to finish the
work ids 6/5. (as per second formula).

Tricks and tips to solve Aptitude problems on Time and Work:

a)If A can finish a work in x days, B can finish the work in y days, then and A and B together takes
time to finish the work is xy/(x+y)
b) If A can finish a work in x days, B can finish the work in y days, C and finish the work in z days then
now together A&B&C finish the job in (x*y*z) /(xy+yz+zx).

2. LCM method for time and work problems


Step 1: Find lcm for all the given days/hours/minutes
Step 2: LCM to be considered as total amount of work to be completed.
Step 3: Then, use the formula given below to solve the problem

Let us look at some example problems on "LCM method for time and work problems".

36
Examples on "LCM method for time and work problems”
Example 2:
A and B together can do a piece of work in 12 days and A alone can complete the work in 21 days. How
long will B alone to complete the same work?
Solution:
Let us find LCM for the given no. of days "12" and "21".
L.C.M of (12, 21) = 84
Therefore, total work = 84 units.
A can do = 84 / 21 = 4 units/day
(A+B) can do = 84 / 12 = 7 units/day
B can do = (A+B) - A = 7 - 4 = 3 units/day
No. of days taken by B alone to complete the same work = 84 / 3= 28 days

Practice Exercise – LCM Method


# Questions: 4 Duration: 10 mints Negative Marking: 0.25 Calculator: Not Allowed

1. A and B can do a work in 15 days. B and C can do it in 30 days. C and A can do the same work in 18 days.
They all work together for 9 days and then A left. In how many days can B and C finish remaining work?
a) 9 days b) 6 days c) 5 days d) 8 days

2. A and B each working alone can do a work in 20 days and 15 days respectively. They started the work
together, but B left after sometime and A finished the remaining work in 6 days. After how many days from the
start, did B leave?
a) 7 Days b) 6 Days c) 5 Days d) 3 Days

3. Andrew and Benford working separately can do a piece of work in 9 and 12 days respectively. If they work for
a day alternately, Andrew beginning, in how many days, the work will be completed?
a) 8 (1/4) b) 10 (1/4) c)11(1/4) d) 15 (1/4) e) None of The Above

4. 2 men and 3 boys can do a piece of work in 10 days while 3 men and 2 boys can do the same work in 8 days.
In how many days can 2 men and 1 boy do the work?
a) 15 (1/2) days b) 10 (1/2) days c) 12 (1/2) days d) 20 (1/2) days e) None of The Above

1. a 2. b 3. b 4. c

37
Shortcut!!
One of the best tricks to solve such problems is by finding the efficiency of workers in percent. If A can do
a job in 2 days then he can do 50% in a day.

Number of days required to Work that can be done per day Efficiency in Percent
complete the work
N 1/n 100/n

1 1/1 100%

2 1/2 50%

3 1/3 33.33%

4 ¼ 25%

5 1/5 20%

6 1/6 16.66%

7 1/7 14.28%

8 1/8 12.5%

9 1/9 11.11%

10 1/10 10%

11 1/11 9.09%

Example 3:
A take 5 days to complete a job and B takes 10 days to complete the same job. In how much time they
will complete the job together?
Solution –
A's efficiency = 20%, B's efficiency = 10%. If they work together they can do 30% of the job in a day. To
complete the job they need 3.33 days.

Example 4: A is twice as efficient as B and can complete a job 30 days before B. In how much they can
complete the job together?
Solution - Let efficiency percentage as x
A's efficiency = 2x and B's efficiency = x
A is twice efficient and can complete the job 30 days before B. So,
A can complete the job in 30 days and B can complete the job in 60 days
A's efficiency = 1/30 = 3.33%

38
B's efficiency = 1/60 = 1.66%
Both can do 5% (3.33% + 1.66% ) of the job in 1 day.
So, they can complete the whole job in 20 days (100/5)

If A is x times as good a workman as B, then

Example 5:
A is twice as good a workman as B and together they finish a piece of work in 18 days. In how many days
will A alone finish the work.
Solution:
if A takes x days to do a work then B takes 2x days to do the same work
= > 1/x+1/2x = 1/18 = > 3/2x = 1/18 = > x = 27 days.
Hence, A alone can finish the work in 27 days.

Practice Exercise – LCM Method


# Questions: 4 Duration: 8 mints Negative Marking: 0.25 Calculator: Not Allowed

1. A is 30% more efficient than B. How much time will they, working together, take to complete a job which
A alone could have done in 23 days?
a) 11 Days b) 13 Days c) 20 (3/17) Days d)None of these

2. Amrinder is twice as good as workman as Baljinder and together they finish a piece of work in 18 days. In
how many days will Amrinder alone finish the work?
a) 29 days b) 28 days c) 27 days d) 26 days e) None of The Above

3.A takes twice as much time as B or thrice as much time to finish a piece of work. Working together they
can finish the work in 2 days. B can do the work alone in?
a) 12 hrs b) 6 hrs c) 3 hrs d) 4 hrs

4. A is thrice as good as workman as B and therefore is able to finish a job in 60 days less than B. Working
together, they can do it in:
a) 20 days b) 22 (1/2) days c) 25 days d) 30 days

1. b 2. c 3. b 4. b

39
MDH Principle
Let M=Men, D=Days, H=Hours, W=Work
M ∝ W (M is directly proportional to W)
D ∝ W; H ∝ W => MDH ∝ W
MDH = kW => MDH/W = k
MDH = k(constant) * W can be used to solve problems where 2 situations in terms of men, days and
hours are given:
M1∗D1∗H1/M2∗D2∗H2=W1/W2,

Conclusion:

M1*D1*H1 = M2*D2*H2
M1*D1*H1*W2 = M2*D2*H2*W1 (when work is added)
M1*D1*H1*W2*E1 = M2*D2*H2*W1*E2 (When efficiency is added)

Example 6:
If 6 men working 5 days for 8 hours each can finish a task. How many men would be needed for 4 days
for 6 hours each to finish half the task?
Answer:
M1∗D1∗H1/M2∗D2∗H2=W1/W2
= 6∗5∗8/M2∗4∗6=1/0.5 => By Solving, we get M2 = 5 men

Practice Exercise
# Questions: 3 Duration: 5 mints Negative Marking: 0.25 Calculator: Not Allowed

1. A piece of work can be done by 16 men in 8 days working 12 hours a day. How many men are needed to
complete another work, which is three times the first one in 24 days working 8 hours a day. The efficiency of
the second group is half that of the first group?
a) 36 b) 42 c) 48 d) 52
2. It is given that 16 men working 18 h a day can build a wall 36 m long, 4 m broad and 24 m high in 20 days.
How many men will be required to build a wall 64 m long, 6 m broad and 18 m high working 12 h a day in 16
days? a) 60 b) 20 c) 30 d) 35
3. 9 men working 7 hours a day can complete a piece of work in 15 days. In how many days can 6
men working for 9 hours a day, complete the same piece of work?
a) 8 days b) 16 days c) 12 days d) None of these

1. c 2. a 3. d

40
Work and Wages by Shortcut Methods

In work and wages two points are important to remember:


 Wages are directly proportional to the work done.
 Wages are inversely proportional to the time taken to complete the work.

Let’s take the more understanding by practicing some examples:

Example 7:
Venky can do a work in 6 days and Himu can do the same work in 7 days working alone. If the total
amount given for this work is Rs.780 when they worked together, what will be the share of Venky?
Solution:
Here Venky and Himu's share of the total will be in the ratio of inverse of their number of days of
completion of the work, that is, in the ratio of 1/6 : 1/7 which is, 7 : 6.
Let us assume the ratio to be 7x:6x, so total is 13x=Rs.780
Thus, x=Rs.60, and Venky's share, 7x=Rs.420.

Example 8:
25 men worked together for 16 days to get an wage of Rs.11500. How many women must work together
for 48 days to receive an wage of Rs.31050, if daily a woman receives half the wage of a man?
Solution:
Wage of a man per day =Rs.11500/25×16=Rs.115/4
Daily wage of a woman being half that of a man, it is, Rs.115/8
So to receive Rs.31050 in 48 days, the number of women working would be,
Number of women =31050/(48×115/8) =31050/(6×115) =5175/115 =1035/23 = 45

Let’s see how much clearance and understanding you have gained! Duration: 5 minutes

1. A, B and C together earn Rs.1350 in 9 days. A and C together earn Rs.470 in in 5 days. B and C together
earn Rs.760 in 10 days. Find the daily earning of C.
a) Rs.20 b) Rs.10 c) Rs.30 d) Rs.50
2. 3 men and 4 boys can earn Rs.756 in 7 days. 11 men and 13 boys can earn Rs.3008 in 8 days. In what time
will 7 men with 9 boys earn Rs.2480?
a) 2/5 b) 5/3 c) 5/6 d) ½
3. A, B and C contract a work for Rs.550. Together, A and B are supposed to do (7/11) of the work. How
much does C get? a) 200 b) 100 c) 150 d) 300

1. a 2.b 3. a

41

1. c 2. a 3. d
Pipes & Cisterns

Here, the whole question revolves around “tank”. Basically, we’ve to find out in how much time
whole tank could be filled or emptied. Also, there can any number of Inlet pipes.

Inlet pipes are responsible for filling the tank. They, basically, bring the water in. The work done by
them is positive. Then we an Outlet pipe, there can be any number of outlet pipes too. Outlet pipes are
responsible for emptying the tank. They, basically, put the water out. The work done by them is
negative.

Important Rules to Remember!

1. If a pipe can fill the tank in ‘x ’ hours then, the part filled in 1 hour = 1/x
2. If a pipe can empty the tank in ‘ y’ hours then, the part emptied in 1 hour = 1/y
3. If a pipe can fill the tank in ‘x ’ hours and another can empty it in ‘y’ hours then, the net part
filled in 1 hour = 1/x – 1/y ; Total time taken to fill such tank = xy/y-x
4. If three such pipes are there where ‘x’, ‘y’ are inlet and ‘z’ is outlet, Total Time

=
5. A pipe can fill the tank in ‘x’ hrs. Due to leak it is filled in ‘y’ hrs, time taken by leak to empty the
tank = xy/y – x hrs
6. If leak time > Inlet pipe then tank will be filled; If leak time < Inlet pipe then tank will be emptied

Example 9:
Pipe A can fill the tank in 20 hours while Pipe B alone can fill it in 30 hours and Pipe C can empty the tank
in 40 hours. If all the pipes are opened together, in how long will the tank be full?
Solutions:
Net part filled in 1 hour= 1/20 + 1/30 – 1/40 (as work done by C is negative)
= 7/120
⇒ Full tank will be full in 120/7 = 17 (1/7) hours.

42
Example 10:
Two pipes P and Q would fill tank in 24 hours and 32 hrs respectively. If both pipes are opened together,
find when the first pipe must be turned off so that the tank may be just filled in 16 hrs?
Solution:
Suppose the pipe P is closed after ‘x’ hours.
Then, P pipe would fill in 1 hr = 1/24 and in x hrs = x / 24
Pipe Q would fill in 1 hour = 1 / 32 and in 16 hrs (as tank is full in 16 hrs) = 16 /32 = 1/2
Pipe P work in ‘x’ hr + Pipe Q work in 16 hrs = 1 (as they complete the 1 unit of work) = x/24 +
16/32 = 1
⇒ x = 12 hours.
Shorter Method

The first pipe should work for =

Practice Exercise
# Questions: 5 Duration: 10 mints Negative Marking: 0.25 Calculator: Not Allowed

1. Three pipes A, B and C can fill cistern in 6 hrs. After working together for 2 hrs, C is closed and A & B fill it in
8 hrs. Then find the time in which cistern can be filled by pipe C.
a) 8 b) 10 c) 14 d) 12

2. Two pipes can separately fill a tank in 20 hrs and 30 hrs respectively. Both the pipes are opened to fill the
tank but when tank is 1/3 full a leak is developed in the tank through which 1/3 of water supplied by both the
tank leak out. What is total time taken to fill the tank?
a) 16 b) 8 c) 12 d) 18

3. Two pipes A and B can fill a tank in 6 hours and 4 hours respectively. If they are opened on
alternate hours and if pipe A is opened first, in how many hours, the tank shall be full ?
a) 3 Hrs b) 5 Hrs c) 7 Hrs d) 10 Hrs

4. There are two pipes which are functioning simultaneously to fill a tank in 12 hours, if one pipe fills the tank
10 hours faster than other then how many hours second pipe will take to fill the tank?
a) 30 Hrs b) 35 Hrs c) 40 Hrs d) 42 Hrs

5. Two pipes P and Q can fill a cistern in 12 minutes and 16 minutes respectively. Simultaneously both the
pipes are opened together, then after how much time Q should be closed so that the tank is full in 9 min?
a) 3.5 min. b) 4 min. c) 4.5 min. d) 4.75 min.

1. d 2. a 3. b 4. a 5. b

43

1. c 2. a 3. d
Review Exercise 8
Seating Arrangement
# Questions: 5 Duration: 10 mints Negative Marking: 0.25 Calculator: Not Allowed

1. In a March Past, seven persons are standing in a row. Q is standing left to R but right to P. O is standing
right to N and left to P. Similarly, S is standing right to R and left to T. Find out who is standing in the middle.
a) P b) Q c) R d) O

2. Mr. A, Miss B, Mr. C and Miss D are sitting around a table and discussing their trades.
(1) Mr. A sits opposite Mr. C.
(2) Miss B sits right to the barber.
(3) The washer man is on the left of the tailor.
(4) Miss D sits opposite Mr. C.
What are the trades of A and B ?
a) Tailor and Barber b) Tailor and Cook c) Barber and Cook d) Washer man and Cook

3. In a pile of reading material, there are novels, story-books, dramas and comics.
Every novel has a drama next to it, every story-book has a comic next to it and there is no story-book next
to novel. If there be a novel at the top and the number of books be 40, the order of the books in the pile is
a) nscd b)csdn c) ndsc d) dncs

4. Five persons A, B, C, D and E are sitting in a row facing you such that D is on the left of C and B is on the
right of E. A is on the right of C and B is on the left of D. If E occupies a corner position, then who is sitting in
the centre ?
a)A b) B c) C d) D
5. Six friends A, B, C, D, E and F are sitting in a closed circle facing the centre. A is facing D. C is between A
and B. F is between E and A. Who is to the immediate left of B?
a) A b) C c) D d) E

1. b 2. b 3. c 4. d 5. b

44
Review Exercise 9
Ranking
# Questions: 10 Duration: 15 mints Negative Marking: 0.25 Calculator: Not Allowed

1. In the following list of numerals, how many 2's are followed by 1's but not preceded by 4?
4 2 1 2 1 4 2 1 1 2 4 4 4 1 2 2 1 2 1 4 4 2 1 4 2 1 2 1 2 4 1 4 2 1 2 4 1 4 6
a) Two b)Three c) Four d) Five
2. How many even numbers are there in the following sequence of numbers which are immediately followed
by an odd number as well as immediately preceded by an even number?
8 6 7 6 8 9 3 2 7 5 3 4 2 2 3 5 5 2 2 8 1 1 9
a) One b)Three c) Five d) Six e) None of these
3. Thirty six vehicles are parked in a parking lot in a single row. After the first car, there is one scooter. After
the second car, there are two scooters. After the third car, there are three scooters and so on. Work out the
number of scooters in the second half of the row.
a) 10 b) 12 c) 15 d) 17
4. In a row of boys, Kapil is eighth from the right and Nikunj is twelfth from the left. When Kapil and Nikunj
interchange positions, Nikunj becomes twenty first from the left. Which of the following will be Kapil's
position from the right ?
a) 8th b) 17th c) 21st d) Can’t be determined
5. Some boys are sitting in a row. P is sitting fourteenth from the left and Q is seventh from the right. If there
are four boys between P and Q, how many boys are there in the row?
a) 25 b) 23 c) 21 d) 19 e) None of these
6. In a class of 60, where girls are twice that of boys, Kamal ranked seventeenth from the top. If there are 9
girls ahead of Kamal, how many boys are after him in rank ?
a) 3 b) 7 c) 12 d) 23 e) 32
7. Three persons A, B and C are standing in queue. There are five persons between A and B and eight persons
between B and C. If there ne three persons ahead of C and 21 persons behind A, what could be the minimum
number of persons in the queue ?
a) 41 b) 40 c) 28 d) 27
8. Ashish leaves his house at 20 minutes to seven in the morning, reaches Kunal`s house in 25 minutes, they
finish their breakfast in another 15 minute and leave for their office which takes another 35 minutes, At what
time do they leave Kunal`s houses to reach their office ?
a) 7.40 a.m. b) 7.20 a.m. c) 7.45 a.m. d) 8.15 a.m. e) 7.55 a.m.
9. A monkey climb s 30 feet before at the beginning of each hour and rests for a while when he slips back 20
feet before he again starts climbing in the beginning of the next hour. If he begins his ascent at 8.00 a.m, at
what time will he first touch a flag at 120 feet from the ground?
a) 4 p.m. b) 5 p.m. c) 6 p.m. d) None of these
10. Standing on a platform, Amit told Sunita that Aligarh was more than ten kilometers but less than fifteen
kilometers from there. Sunita knew that it was more than twelve but less then fourteen kilometers from
there. If both of them were correct, which of the following could be the distance of Aligarh from the
platform?
a) 11 km b) 12 km c) 13 km d) 14 km e) 15km

45
1. c 2. e 3. c 4. b 5. a 6. c 7. c 8. b 9. c 10. c
Review Exercise 10
1. c 2. e 3. c 4. b 5. a
Ages
6. c 7. c 8. b 9. c 10. c
# Questions: 5 Duration: 10 mints Negative Marking: 0.25 Calculator: Not Allowed

Q1. Present ages of Sameer and Anand are in the ratio of 5 : 4 respectively. Three years hence, the ratio of
their ages will become 11 : 9 respectively, What is Anand’s present age in years?
a. 22 b. 24 c. 26 d. 30 e. None of these
Q2. One year ago, Promila was four times as old as her daughter Sakshi. Six years hence, Promila’s age will
exceed her daughter’s age by 9 years. The ratio of the present ages of Promila and her daughter is:
a. 13 : 4 b. 4 : 13 c. 5 : 20 d. 20 : 5 e. None of these
Q3. A father said to his son, "I was as old as you are at present at the time of your birth." If the father's
age is 38 years now, the son’s age five years back was:
a.14 1. b.19b c.38 d.40
2. a e. none of these3. a 4. c 5. b
Q4. Ayesha's father was 38 years of age when she was born while her mother was 36 years old when her
brother four years younger to her was born. What is the difference between the ages of her parents?
a.2 years b. 4 years c.6 years d.8 years
Q5. Father is aged three times more than his son Ronit. After 8 years, he would be two and a half times of
Ronit's age. After further 8 years, how many times would he be of Ronit's age?
a. 1.5 b. 2 c. 2.5 d. 3 e. None of these

1. b 2. a 3. a 4. c 5. b

46
Number System

Factors and Divisibility of a Number

How good you are in Factors??

Given an integer N, there is a simple way to find the total number of its factors. The main
tool for the feat is the prime number decomposition theorem.
These are certain basic formulas pertaining to factors of a number N, such that,
N= p^a*q^b*r^c
Where, p, q and r are the prime factors of the number N. a, b and c are non-negative
powers/ exponents.

1. Number of factors of N = (a+1)(b+1)(c+1)


2. Number of odd factors of N = product of only odd numbers power increased by 1.
3. Number of even factors of N = Total factors – odd factors
4. Number of prime factors of N = addition of powers=a+b+c.
5. Product of factors of N = N No. of factors/2
6. Sum of factors of N = (p0+p1+...+pa) (q0+ q1+....+qb) (r0+r1+...+rc)

Example- Consider the number 120. Find the following for n:


1. Sum of factors. 2. Number of factors. 3. Product of factors.
4. Odd factors. 5. Even factors. 6. Prime factors.
Solution- The prime factorization of 120 is 2^3 x 3^1 x 5^1. By applying the formulae,
1. Sum of factors = [(20+21+22+23)(30+31)(50+51)]= 1560
2. Number of factors = (3+1)(1+1)(1+1) = 16
3. Product of factors = 120(16/2) = 1208
4. Odd factors= (1+1)*(1+1) = 4
5. Even factors=16-4 = 12
6. Prime Factors= 3+1+1 = 5

47
Divisibility Tests Example
A number is divisible by 2, if the last digit is 0, 2, 4, 6 168 is divisible by 2 since the last digit is 8.
or 8.
A number is divisible by 3, if the sum of the digits is 168 is divisible by 3 since the sum of the digits is 15
divisible by 3. (1+6+8=15), and 15 is divisible by 3.
A number is divisible by 4, if the number formed by 316 is divisible by 4 since 16 is divisible by 4.
the last two digits is divisible by 4.
A number is divisible by 5, if the last digit is either 0 195 is divisible by 5 since the last digit is 5.
or 5.
A number is divisible by 6, if it is divisible by 168 is divisible by 6 since it is divisible by 2 AND it is
2 AND it is divisible by 3. divisible by 3.
A number is divisible by 8, if the number formed by 7,120 is divisible by 8 since 120 is divisible by 8.
the last three digits is divisible by 8.
A number is divisible by 9, if the sum of the digits is 549 is divisible by 9 since the sum of the digits is 18
divisible by 9. (5+4+9=18), and 18 is divisible by 9.
A number is divisible by 10, if the last digit is 0. 1,470 is divisible by 10 since the last digit is 0.

Practice Set A
Q1. Find the following for the number 84?
1. Number of odd factors. 2. Number of even factors.
A. 4,8 B. 5,5 C. 8,12 D. 7,9
Q2. How many factors of 1200 are odd integers?
A. 6 B. 8 C. 12 D. 22
Q3. Find the total no of prime factors in 4^11 x 7^5 x 11?
A. 17 B. 27 C. 28 D. 30
Q4. Find the sum of factors of 18?
A. 6 B. 13 C. 39 D. 35
Q5. Find the odd value of a if a number 34a6 divisible by 3?
A. 3 B. 5 C. 9 D. 7
Q6. What is the value of M and N respectively? If M39048458N is divisible by 8 and 11; where M and N are single
digit integers?
A. 7, 8 B. 8, 6 C. 6, 4 D. 5, 4
Q7. Find the value or values of a and b if a given number 624ab is divisible by 5 and 8 both?
A. 0,0 B. 4,0 C. 8,0 D. All of them

1. A 2. A 3. C 4. C 5. B 6. C 7. D

48
How good you are in Factorial??
The factorial function (symbol: !) means to multiply a series of descending natural numbers.
An older notation for the factorial is
N!=N(N-1)(N-2)………1.
4!=4*3*2*1=24
Note- 0!=1 and 1!=1.

Trailing Zeros or Ending zeros in N!


For example, 5!=120. So, it has only one zero in end.

Rule for finding trailing zeros- Divide the given number by the powers of 5 till it divisible by powers of 5.It
means numerator is greater or equal to denominator.
N/5 + N/5^2 + N/5^3………….N>= 5^n
Here we take only quotient of it.

Example- Find the trailing zeros in 102!


DFDG
102/5 + 102/25 = 20+4=24
(Here 100/125 is not possible, so divide by 5’s powers till it is less or equal to number)
So, 102! Have 24 zeros.

Highest power of a number in a factorial or in a product

Highest power of p (prime number) in N! is [N/p] +[N/p^2] +[N/p^3]+……..[N/p^n] till N>=p^n.


Take only quotient of these divisions.
Eg.1- Highest power of 2 in 50!? 50/2 +50/4 +50/8+50/16 +50/32=25+12+6+3+1=47
Eg.2- Highest power of 6 in 20!?
6 is a composite number. To find the highest power of composite number write it into prime factorization, i.e.,
6=2x3.Now, find the highest power of 2 and 3 in 20!.
Highest power of 2 is= 20/2+20/4+20/8+20/16=10+5+2+1=18
Highest power of 3 is=20/3+20/9=6+2=8
Highest power of 6 is the least value which of individual highest powers. Here values are 18 and 8. So, the
highest power of 6 is 8.

Highest power of p^a in N! is [N/p +N/p^2+N/p^3+………N/p^n] / a


Here a is natural number and p is prime.
Eg.- Highest power of 72 in 50!
72=8x9=2^3 x 3^2
Highest power of 2^3 =[50/2+50/4+50/8+50/16+50/32]/3=[25+12+6+3+1]/3=15
Highest power of 3^2=[50/3+50/9+50/27]/2=[16+5+1]/2=11
So, the highest power of 72 is 11.

49
Practice Set B
Q1. Find the number of trailing zeroes in the expansion of 23!
A. 5 B. 4 C. 20 D. 21
Q2. Find the number of trailing zeroes in the expansion of 1000!
A. 250 B. 300 C. 249 D. 245
Q3. Find the number of zeros in 2*3*4*5…………*125?
A. 30 B. 35 C. 38 D. 31
Q4. Find the number of factors of 6!?
A. 25 B. 30 C. 35 D. 32
Q5. Find the highest power of 24 in 150!?
A. 48 B. 72 C. 58 D. 45
Q6. Find the highest power of 30 in 40!?
A. 12 B. 10 C. 8 D. 9

1.B 2.C 3.D 4.B 5.A 6.D

50
Unit Digits
Unit digit of product- Just multiply last digits of each number.
Eg.- 121x76x528x172= 1x6x8x2=96= 6 is unit digit here.
Unit digit of powers- Either use cyclicity of number or use simple method.

2 3 4 5 6 7 8 9
21=2 31=3 41=4 51=5 61=6 71=7 81=8 91=9
22=4 32=9 42=6 52=5 62=6 72=9 82=4 92=1
23=8 33=7 43=4 53=5 63=6 73=3 83=2 93=9
24=6 34=1 44=6 54=5 64=6 74=1 84=6 94=1
25=2 35=3 45=4 55=5 65=6 75=7 85=8 95=9
26=4 36=9 46=6 56=5 66=6 76=9 86=4 96=1
27=8 37=7 47=4 57=5 67=6 77=3 87=8 97=9

Eg.- Find the unit digit in 2^49?


We know in case of 2, it repeats itself after a cycle of 4 . We will divide 49 by 4
49/4 remainder is 1
We write it as 2^49= 2^1= 2. That means the unit digit in the 2^49 is 2.

Rule for numbers ending in digits 0 or 1 or 5 or 6 :- unit digits of that numbers are same as there last digits ending
in 0 or 1 or 5 or 6 whatever the power is.
Eg.- (235)^27= unit digit 5
(126)^344= unit digit 6

Rule for numbers ending in digits 2,3,4,7,8 and 9 :- Divide the power by 4 find the remainder. Make that remainder
to the power of last digit of the number will give us the unit digit.
Note- if remainder is 0 (power completely divisible by 4) take remainder as 4 not 0.

Eg.1- (327)^22
22/4 =Rem(2)
Last digit is 7. Make remainder 2 to power of 7=7^2=49
So , 9 is a unit digit.

Eg.2- (28)^36
36/4=Rem(0). Here take remainder as 4.
Last digit is 8. Then, 8^4 = 64x64 = 4x4 =16.
So, unit digit is 6

51
Practice Set C1

Q1. The digit in the unit place of the number 7295 X 3158 is
A. 7 B. 2 C. 6 D. 4

Q2. What is the unit digit in the product (365 x 659 x 771)?
A. 1 B. 2 C. 4 D. 6

Q3. The unit digit of is:


A. 1 B. 3 C. 5 D. 7

Q4. The unit digit of is:


A. 2 B. 4 C. 6 D. 8

1.A 2.C 3.B 4.B 5.A 6.D


Q5. The unit digit of is:
A. 2 B. 4 C. 6 D. 8

Q6. Find the unit digit of (23)^25! ?


A. 0 B. 2 C. 3 D. 1

1.A 2.C 3.B 4.B 5.A 6.D

52
Remainders
Remainder Theorem:- Dividend =Divisor x Quotient + Remainder
When dividend is of the form a n + b n or a n - b n :

When f(x) = a + bx + cx 2 + dx 3 +... is divided by x - a

Eg.- What is the remainder when the product 1998 × 1999 × 2000 is divided by 7?
Find the individual remainders of 1998, 1999, and 2000 are divided by 7 are 3, 4, and 5 respectively. Hence, the
final remainder is the remainder when the product 3 × 4 × 5 = 60 is divided by 7.So, the final re mainder is 4.

Fermat’s theorem-This theorem is stated in the following form: if p is a prime and a is an integer co-prime to
p, then a^(p−1) − 1 will be evenly divisible by p. In other words, [a^(p-1)]/p gives remainder 1.

Eg.- Find the remainder when 72^40 divide by 41?


Answer: So here we see that 41 is a prime number, so we will target Fermat’s little theorem instead of Euler’s
theorem.
Again 72 and 41 are co-prime. so we can apply our little theorem in this problem easily.
–> remainder [72^40/41] = 1.

Wilson’s Theorem- This theorem state that for a prime number p , (p-1)! Divide by p , then the remainder is p-1.

Eg.- Find the remainder when 16! is divided by 17.


16! = (16! + 1) -1 = (16! + 1) + 16 - 17
Every term except 16 is divisible by 17 in the above expression. Hence the remainder = the remainder obtain ed
when 16 is divided by 17 = Rem(16).

53
Practice Set C2
Q1. What is the remainder when 13x14x16 divide by 6?
A. 2 B. 0 C. 6 D. 7
Q2. On dividing a number by 357, we get 39 as remainder. On dividing the same number 17, what will be the
remainder?
A. 0 B. 3 C. 5 D. 11
Q3. What is the remainder when 3 444 + 4 333 is divided by 5?
A. 0 B. 1 C. 3 D. 4
Q4. What is the remainder when (5555) 2222 + (2222) 5555 is divided by 7?
A. 2 B. 4 C. 0 D. 1
Q5. Find the remainder when 7 52 is divided by 2402.
A. 1 B. 0 C. 2400 D.2401
Q6. What is the remainder when x 3 + 2x 2 + 5x + 3 is divided by x + 1?
A. 1 B. -1 C. 12 D. 33
Q7. If 2x 3 - 3x 2 + 4x + c is divisible by x - 1, find the value of c?
A. 10 B. -5 C. 12 D. -3

1.A 2.C 3.A 4.C 5.D 6.B 7.D

HCF and LCM


Important Terms:
1) Factors: Factor is a number which exactly divides other number.
2) Multiple: A number is said to be multiple of another number, when it is exactly divisible by other number.
3) Common multiple: A common multiple of two or more numbers is a number which is exactly divisible by each of
them.
4) H.C.F/G.C.F: (Highest Common Factor / Greatest Common Factor). H.C.F of two or more numbers is the greatest
number which divides each number exactly.
5) L.C.M.: (Lowest common multiple). The least number exactly divisible by each one of the given numbers is called
least common multiple.
Tips and Tricks:
1) H.C.F. and L.C.M. of Fractions
H.C.F. of Numerator
a) H.C.F. =
L.C.M. of Denominator

L.C.M. of Numerator
b) L.C.M. =
H.C.F. of Denominator
2) Product of two numbers = Product of their H.C.F. and L.C.M.
This condition is only true for two given numbers. If H.C.F. and L.C.M. of three or more numbers are given, then
this rule is not applicable.

54
Methods to Find H.C.F. of Given Numbers

55
Practice Set D
Q1. H.C.F. of 513, 1134 and 1215 is _____________
A. 18 B. 27 C. 33 D. 36
Q2. Find the largest number of 4-digits divisible by 12, 15 and 18.
A. 9900 B. 9750 C. 9450 D. 9000
Q3. Find L.C.M. of 1.05 and 2.1
A. 1.3 B. 1.25 C. 2.1 D. 4.30
2 16 8
Q4. Calculate H.C.F. of , ,
3 81 9
A. 2/9 B. 8/3 C. 2/81 D. 3/16
Q5. H.C.F. of two numbers is 13. If these two numbers are in the ratio of 15: 11, then find the numbers.
A. 230, 140 B. 215, 130 C. 195, 143 D. 155, 115
Q6. Find the greatest number, which on dividing 1657 and 2037 leaves remainders 6 and 5 respectively.
A. 127 B. 132 C. 114 D. 108
Q7. Five bells commence tolling together and toll at intervals 2, 4, 6, 8 and 10 seconds respectively. Find
in 40 minutes, how many times do they toll together?
A. 8 times B. 19 times C. 21 times D. 30 times
Q8. Find the least number which when divided by 5, 6, 7 and 8 leaves a remainder 3, but when divided by
9 leaves no remainder.
A. 1963 B. 2523 C. 1683 D. 1536
Q9. Find the greatest number that will divide 43, 91 and 183 so as to leave the same remainder in each
case.
A. 4 B. 7 C. 9 D. 13
Q10. The traffic lights at three different road crossings change after every 40 sec, 72 sec and 108 sec
respectively. If they all change simultaneously at 5 : 20 A.M., then find the time at which they will change
simultaneously.
A. 5 : 28 B. 5 : 30 C. 5 : 38 D. 5 : 40

1.B 2.A 3.C 4.C 5.C

6.A 7.C 8.C 9.A 10.B

56
A.P. and G.P.
An Arithmetic Progression (A.P.) is a sequence in which the difference between any two consecutive terms is
constant. Let a = first term, d = common difference
Then, nth term an = a + (n-1)d

AM (Arithmetic mean): If a, b, c are in AP then the arithmetic mean is given by b = (a+c)/2

Inserting AM: To insert k means between a and b the formula for common difference is given by
d=(b-a)/k+1
For Example: Insert 4 AM’s between 4 and 34
d= (34-4)/4+1= 30/5= 6
∴ The 4 AM are 4+6=10, 10+6=16, 16+6=22 , 22+6=28

Geometric Progression: Geometric sequences are powers rk of a fixed number r, such as 2k and 3k. The
general form of a geometric sequence is

Sum of G.P.= a(1-r^n)/(1-r)

GM (Geometric mean): If a, b, c are in GP Then the GM is given by b = √ac


Note: 1. AM>GM>HM 2. GM^2=AMxHM

Inserting GM: To insert k means between a and b the formula for common ratio is given by
r = (b/a)^(1/(k+1))
For example: Insert 4 GM’s between 2 and 486
r = (486/2)^(1/(4+1))= (243)^(1/5)= 3
∴ The 4 GM are 2x3 = 6, 6x3 = 18, 18x3 = 54 , 54x3 = 162.

57
1.C 2.A 3.B
Practice
4.A
Set
5.D
E 6.C 7.D 8.B

Q1. Find the number of terms in the series 8, 12, 16, . . .72
A. 10 B. 12 C. 17 D. 16

Q2. The sum of third and ninth term of an A.P is 8. Find the sum of the first 11 terms of the progression.
A. 44 B. 22 C. 19 D. None of the above

Q3. Find 4 + 7 + 10 + 13 + 16 + . . . up to 20 terms


A. 600 B. 650 C. 540 D. 454

Q4. Find 5th term in the series 5, 15, 45, ...


A. 405 B. 345 C. 450 D. 340

Q5. Given A = 265 and B = (264+263+262+...+20)


A. B is 264 larger than A B. A and B are equal
C. B is larger than A by 1 D. A is larger than B by 1

Q6. How many 2-digit positive integers are divisible by 4 or 9?


A. 32 B. 22 C. 30 D. 34

Q7. If a rubber ball consistently bounces back ⅔ of the height from which it is dropped, what fraction of
its original height will the ball bounce after being dropped and bounced four times without being stopped
A. 16/17 B. 4/81 C. 37/81 D. 16/81
Q8. If log 2, log (2x -1) and log (2x + 3) are in A.P, then x is equal to ____
A. 5252 B. log25 C. log32 D. 32

1.C 2.A 3.B 4.A 5.D 6.C 7.D 8.B

58
Non Verbal Reasoning

Review Exercise 11
# Questions: 20 Duration: 15 mints Negative Marking: 0.25 Calculator: Not Allowed

1. Select a figure from amongst the Answer Figures which will continue the same series as established by the five
Problem Figures.
Problem Figures: Answer Figures:

(A) (B) (C) (D) (E) (1) (2) (3) (4) (5)
a.1 b.2 c.3 d.4 e.5
2. Select a figure from amongst the Answer Figures which will continue the same series as established by the five
Problem Figures.
Problem Figures: Answer Figures:

(A) (B) (C) (D) (E) (1) (2) (3) (4) (5)
a.1 b.2 c.3 d.4 e.5
3. Select a suitable figure from the Answer Figures that would replace the question mark (?).
Problem Figures: Answer Figures:

(A) (B) (C) (D) (1) (2) (3) (4) (5)


a.1 b.2 c.3 d.4 e.5
4. Choose the figure which is different from the rest.

(1) (2) (3) (4) (5)


a.1 b.2 c.3 d.4 e.5

59
5. Choose the figure which is different from the rest.

(1) (2) (3) (4) (5)


a.1 b.2 c.3 d.4 e.5
6. Choose the alternative which is closely resembles the mirror image of the given combination.

a.1 b.2 c.3 d.4


7. Choose the alternative which is closely resembles the water-image of the given combination.

a.1 b.2 c.3 d.4


8. Find out the alternative figure which contains figure (X) as its part.

(X) (1) (2) (3) (4)


a.1 b.2 c.3 d.4
9. Identify the figure that completes the pattern.

(X) (1) (2) (3) (4)


a.1 b.2 c.3 d.4

60
10. Select a suitable figure from the four alternatives that would complete the figure matrix

a.1 b.2 c.3 d.4


11. Find out from amongst the four alternatives as to how the pattern would appear when the transparent
sheet is folded at the dotted line.

(X) (1) (2) (3) (4)


a.1 b.2 c.3 d.4
12. Find out from amongst the four alternatives as to how the pattern would appear when the transparent
sheet is folded at the dotted line.

(X) (1) (2) (3) (4)


a.1 b.2 c.3 d.4
13. Choose a figure which would most closely resemble the unfolded form of Figure (Z).

a.1 b.2 c.3 d.4

14. Choose a figure which would most closely resemble the unfolded form of Figure (Z).

a.1 b.2 c.3 d.4

61
15. Choose the set of figures which follows the given rule.
Rule: Closed figures become more and more open and open figures become more and more closed.

a.1 b.2 c.3 d.4


16. Select the figure which satisfies the same conditions of placement of the dots as in Figure-X.

a.1 b.2 c.3 d.4


17. Select the alternative which represents three out of the five alternative figures which when fitted into each
other would form a complete square.

a.145 b.245 c.123 d.234


18. Find out which of the figures (1), (2), (3) and (4) can be formed from the pieces given in figure (X).

a.1 b.2 c.3 d.4


19. Find out which of the figures (1), (2), (3) and (4) can be formed from the pieces given in figure (X).

a.1 b.2 c.3 d.4


20. Find the number of triangles in the given figure.

a.8 b.12 c.13 d.14 e.10

1.C 2.E 3.D 4.A 5.D 6.C 7.A 8.A 9.B 10.C
11.C 12.B 13.D 14.D 15.B 16.D 17.B 18.A 19.A 20.D

62
Time, Speed & Distance

Average Speed = Total Distance/Total Time


1. Average Speed = (a + b)/2
Applicable when one travels at a speed for half the time and speed b for other half of the time. In this case,
average speed is the arithmetic mean of the two speeds.
2. Average Speed = 2ab/ (a + b)
Applicable when one travels at a speed for half the distance and speed b for other half of the distance. In this
case, average speed is the harmonic mean of the two speeds. On similar lines, you can modify this formula for
one-third distance.
3. Average Speed = 3abc/(ab + bc + ca)
Applicable when one travels at a speed for one-third of the distance, at speed b for another one-third of the
distance and speed c for rest of the one-third of the distance.
Note that the generic Harmonic mean formula for n numbers is
Harmonic Mean = n/(1/a + 1/b + 1/c + …)
4. You can also use weighted averages. Note that in case of average speed, the weight is always ‘time’. So in
case you are given the average speed, you can find the ratio of time as
t1/t2 = (a – Avg)/(Avg – b)

63
Example 1:-
Myra drove at an average speed of 30 miles per hour for the first 30 miles of a trip & then at an average speed of 60
miles/hr for the remaining 30 miles of the trip. If she made no stops during the trip what was her average speed in
miles/hr for the entire trip?
Solution:-
Here, distance for which Myra traveled at the two speeds is same.
Average Speed = 2ab/(a+b) = 2*30*60/(30 + 60) = 40 mph

Practice Exercise – Average Speed


1. A person crosses a 600 m long street in 5 mints. What is his speed in km/h:
a.3.6 b. 7.2 c. 8.4 d. 10

2. If a man runs at 3m/s, how many kilometers does he run in 1 h 40 min?


a. 18 km b. 12 km c. 20 km d. 22 km

3. A man on tour travels first 160km at 64 km/h and the next 160 km at 80 km/h. The average speed for
the first 320 km of the tour is:
a.35.55 km/h b. 36 km/h c. 71.11 km/h d. 71km/h

4. What is the average speed if a person travels at the speed of 20km/h and 30 km/h for the equal
interval of time?
a. 20 km/h b. 25 km/h c. 22 km/h d. 26 km/h

1. B 2. a 3. c 4. b

Relative Speed

Case 1:
Two bodies are moving in opposite directions at speed V1 & V2 respectively. The relative speed is defined
as, Vr=V1+V2

Case 2:
Two bodies are moving in same directions at speed V1 & V2 respectively. The relative speed is defined
as, Vr=|V1−V2|

64
Example 1:-A train is running at a speed of 90 km/hr. if it crosses a pole in just 10 second, what is the
length of the train?
Solution: Speed of the train = 90 km/hr. Speed of the train = 90 × 5/18 m/sec = 25 m/sec. Time taken by
the train to cross the pole = 10 seconds. Therefore, length of the train = 25 m/sec × 10 sec = 250 m
Example 2: A train 165 m long is running at the speed of 60 km/hr. In what time will it pass a man who is
running at the speed of 6 km/hr in the same direction in which the train is moving?
Solution:- Man moving in the same direction of the train
Speed of train relative to the man = (60 – 6) km/hr = 54 km/hr
= (54 × 5/18) m/sec = 15 m/sec
Time taken by the train to cross a man = distance/speed
= length of train/speed of train relative to man
= 165 m/15 m/sec= 11 sec
Example 3:-Two trains 130 m and 140 m long are running on parallel tracks in the same direction with a
speed of 68 km/hr and 50 km/hr. How long will it take to clear off each other from the moment they
meet?
Solution:-Relative speed of trains = (68 – 50) km/hr
= 18 km/hr = 18 × 5/18 m/sec = 5 m/sec
Time taken by the train to clear off each other = sum of length of trains/relative speed of trains
= (130 + 140)/5 sec = 270/5 sec = 54 sec
Example 4:-Two trains 163 m and 187 m long are running on parallel tracks in the opposite directions
with a speed of 47 km/hr and 43 km/hr in. How long will it take to cross each other?
Solution:- Relative speed of train = (47 + 43) km/hr = 90 km/hr
= 90 × 5/18 m/sec = 25 m/sec
Time taken by the two trains to cross each other = sum of length of trains/relative speed of trains
= (163 + 187)/25 sec = 350/25 sec= 14 sec
Therefore, the two trains crossed each other in 14 seconds.

Practice Exercise – Relative Speed


1. Two boys starting from the same place walk at the rate of 5 kmph and 5.5 kmph resp. What time will they
take to be 8.5 km apart, if they walk in the same direction?
a.17 hr b. 15 hr c. 17.5 hr d. 16.5 hr
2. A thief steals a car at 2.30 pm and drives it at 60 kmph. The theft is discovered at 3 pm and the owner sets
off in another car at 75 kmph, when he will overtake the thief:
a. 4 pm b. 5 pm c. 4.30 pm d. 5.15 pm
3. In covering distance the speed of A and B are in the ratio of 3:4. A takes 20 mints more than B to reach the
destination. The time taken by A to reach the destination is:
a. 1(1/3) hr b. 2 hrs c. 2(2/3)hrs d. 2 hr 05 mints
4. In covering a distance of 30 km, Abhay takes 2 hrs more than Sameer. If Abhay doubles his speed, then he
would t take 1 hr less than Sameer. Then the speed of Abhay is:
a. 5 kmph b. 6.25 kmph c. 6 kmph d. 7.5 kmph

1. a 2. b 3. a 4. a

65
TRAINS

The following things need to be kept in mind while solving the train related problems.
1. When the train is crossing a moving object, the speed has to be taken as the relative speed of the train
with respect to the object.
2. The distance to be covered when crossing an object, whenever train crosses an object will be equal to:
Length of the train + Length of the object

Example 1:- A 180m long train is running at 54 Kmph. How much time it will take to cross a platform of
120m long?
Solution:-
Formula: D = S × T
180 = 54 Kmph × Time
180= 54×5/18 × Time
Time = 20 Sec

Example 2:- Two, trains, one from Howrah to Patna and the other from Patna to Howrah, start
simultaneously. After they meet, the trains reach their destinations after 9 hours and 16 hours
respectively. The ratio of their speeds is:
Solutions:-Let us name the trains as A and B.
Then, (A's speed) : (B's speed) = √b : √a = √16 : √9 = 4 : 3.

Practice Exercise -Trains


1. A train passes a station platform in 36 seconds and a man standing on the platform in 20 seconds. If the speed
of the train is 54 km/hr, what is the length of the platform?
a. 120 m b. 240 m c. 300 m d. None of these

2. A jogger running at 9 kmph alongside a railway track in 240 metres ahead of the engine of a 120 metres long
train running at 45 kmph in the same direction. In how much time will the train pass the jogger?
a. 3.6 sec b. 18 sec c. 36 sec d. 72 sec

3. It takes 8 hrs for 600 km journey, if 120 km is done by train and the rest by car. It takes 20 mints more, if 200km
is done by train and the rest by car. The ratio of the speed of the train to that of the car is:
a. 2:3 b. 3:2 c. 3:4 d. 4:3

1. b 2. c 3.c

66
BOATS & STREAMS

Let, U be the velocity of boat in still water & V be the velocity of the stream

UPSTREAM: While moving in upstream, distance covered, distance covered, S= (U−V)T


Downstream: In case of downstream, distance covered, distance covered, S= (U+V) T

Example 1:- If a man's rate with the current is 15 km/hr and the rate of the current is 11⁄2 km/hr, then
his rate against the current is?
Solution:- Speed downstream = 15 km/hr
Rate of the current= 11⁄2 km/hr
Speed in still water = 15 - 11⁄2 = 131⁄2 km/hr
Rate against the current = 131⁄2 km/hr - 11⁄2 = 12 km/hr

Example 2:- A boat goes 8 km upstream in 24 minutes. The speed of stream is 4 km/hr. The speed of
boat in still water is?
Solution: Speed upstream =8/(24/60)=8/(24/60) = 20 km/hr
Speed of the stream = 4 km/hr
speed of boat in still water = (20+4) = 24 km/hr

Example 3:- Find the average speed of a boat in a round trip between two places 18 km apart. If the
speed of the boat in still water is 9km/h and the speed of the river is 3km/h?
Solution: Average speed = upstream * downstream / man’s speed in still water
Average speed = 6 * 12 / 9 = 8km/h
Short Trick: Average speed = upstream * downstream / man’s speed in still water

Practice Exercise – Boats & Streams


1.A motorboat whose speed in still water is 10km/h went 91 km downstream and then returned to its
starting point. Calculate the speed of the river flow if the round trip took a total of 20 h.
a. 3 km/h b. 4 km/h c. 6 km/h d. 8 km/h
2 A steamer goes downstream from one point to the other in 4 hrs. It covers the same distance upstream
in 5 hrs. If the speed of the stream is 2 km/hr, the distance between the two points is?
a.50 b.60 c.70 d.80
3. Speed of a boat in standing water is 14 kmph and the speed of the stream is 1.2 kmph. A man rows to a
place at a distance of 4864 km and comes back to the starting point. The total time taken by him is?
a.700 hours b.350 hours c.1500 hours d.900 hours

1. a 2. d 3. a

67
RACES

1. Races: A contest of speed in running, riding, driving, sailing or rowing is called a race.
2. Race Course: The ground or path on which contests are made is called a race course.
3. Starting Point: The point from which a race begins is known as a starting point.
4. Winning Point or Goal: The point set to bound a race is called a winning point or a goal.
5. Winner: The person who first reaches the winning point is called a winner.
6. Dead Heat Race: If all the persons contesting a race reach the goal exactly at the same time, the
race is said to be dead heat race.
7. Start: Suppose A and B are two contestants in a race. If before the start of the race, A is at the
starting point and B is ahead of A by 12 metres, then we say that 'A gives B, a start of 12 metres'.
To cover a race of 100 metres in this case, A will have to cover 100 metres while B will have to cover
only (100 - 12) = 88 metres.
In a 100 m race, 'A can give B 12 m' or 'A can give B a start of 12 m' or 'A beats B by 12 m' means that
while A runs 100 m, B runs (100 - 12) = 88 m.
8. Games: 'A game of 100, means that the person among the contestants who scores 100 points first is
the winner'.
If A scores 100 points while B scores only 80 points, then we say that 'A can give B 20 points.

EXAMPLE 1:- In a 500 m race, the ratio of the speeds of two contestants A and B is 3: 4. A has a start of
140 m. Then, A wins by?
Solution:- To reach the winning post ,A will have to cover a distance of (500 - 140)m, i.e., 360 m. While A
covers 3 m, B covers 4 m.
While A covers 360 m, B covers (4/3)*360 = 480m
Thus, when A reaches the winning post, B covers 480 m and therefore remains 20 m behind.
A wins by 20 m.
Example 2:- In 100 m race, A covers the distance in 36 seconds and B in 45 seconds. In this race A beats
B by?
Solution:- Distance covered by B in 9 sec. = (100/45)*9 = 20m
A beats B by 20 metres.

Practice Exercise - Races


1. A can run 224 metre in 28 seconds and B in 32 seconds. By what distance A beat B?
a.20 metre b. 25 metre c .30 metre d.28 metre
2. A and B take part in 100 m race. A runs at 5 kmph. A gives B a start of 8 m and still beats him by 8
seconds. The speed of B is?
a. 4.4 kmph b. 4.25 kmph c. 4.14 kmph d. 5.15 kmph
3. In a km race A can beat B by 100 m and B can beat C by 60 m. In the same race A can beat C by:-
a. 144 m b.138 m c.149 m d.154 m

1. d 2.c 3.d

68
CIRCULAR MOTION

FIRST MEETING OF THREE BODIES ON CIRCULAR PATH: In case when three or more bodies start moving
simultaneously from the same point on the circumference of the circle, in the same direction around the
circle, they will first meet after a time given by LCM of the times that the fastest runner takes in totally
overlapping each of the slower runners.

FIRST MEETING AT THE STARTING POINT: In case when three or more bodies start moving
simultaneously from the same point on the circumference of the circle, in the same direction around the
circle , they will meet again at the starting point after a time calculated by taking LCM of the times that
each of the bodies takes to complete on full round.

Practice Exercise – Circular Motion


DIRECTION (Q.1-2) Three friends Mohit, Divya and Rajat run along a circular park with a speed of 8
km/h, 5 km/h and 3 km/h respectively. The circumference of the park is 500 m.
1. When will they all be together again for the first time?
a. 25 min b. 30 min c. 40 min d. 20 min
2. When will they all be together again for the first time at the starting point?
a. 25 min b. 30 min c. 40 min d. 20 min

1. b 2. b

69
Review Exercise 12
Data Sufficiency
# Questions: 5 Duration: 10 mints Negative Marking: 0.25 Calculator: Not Allowed

Directions (Q. 1-3): Each of the questions below consists of a question and two statements numbered I and II
given below it. You have to decide whether the data provided in the statements are sufficient to answer the
question. Read both the statements and give answer
A) if the data in statement I alone are sufficient to answer the question while the data in statement II alone are
not sufficient to answer the question.
B) if the data in statement II alone are sufficient to answer the question while the data in statement I alone are
not sufficient to answer the question.
C) if the data either in statement I alone or in statement II alone are sufficient to answer the question.
D) if the data even in both statements I and II together are not sufficient to answer the question.
E) if the data in both statements 1 and II together are necessary to answer the question.
1. Which direction is Shashidhar facing?
Statements: I. In the early morning Shashidhar was standing in front of a puppet and the shadow of the puppet
was falling to the right of Shashidhar.
II. In the early morning Shashidhar was standing on the ground. His shadow was falling behind him when he
turned to his left.
2. Who among A, B, C, D and E teaches History?
Statements: I. Each one of them teaches only one subject. B teaches Mathematics, while E teaches Science. A or
C does not teach Geography. A or D does not teach English.
II. C and E are teachers of English and Science respectively and A is the teacher of Mathematics.
3. In a row of boys facing South who is immediate left to Ramakant?
Statements: I. Suresh is immediate right to Chandrakant, who is fourth to the right of Ramakant.
II. Suresh is third to the right of Ramakant and Naresh is second to the right of Suresh.
Directions (Q. 4-5): Each of the questions below consists of a question and two or three statements given below
it. You have to decide whether the data provided in the statements are sufficient to answer the question
4. Who is the uncle of L?
Statement: A) P, brother of M, is father of L; M is father of S. B) R is father of L’s cousin
A. A alone is sufficient B. B alone is sufficient
C. Either A alone or B alone is sufficient D. Both A and B together are not sufficient
E. Both A and B together are necessary
5. Who is to the North-East of R?
Statement: A) S is to the South-East of N, who is to the South West of P, who is to the North of Q.
B) T is to the North-West of Q, who is to the South of P.
C) R, who is to the North of S, is midway between N and Q, N being to the west of R.
A. All A, B, C together are required B. Only A and C together are sufficient
C. Only B and C together are sufficient D. Either A and C together or B and C together are
sufficient E. None of these

1. C 2. A 3. D 4. C 5. D

70
Permutation & Combination

Important Facts & Formulae


Factorial Notation:
Let n be a positive integer. Then factorial n denoted by n! is defined as n! = n(n-1)(n-2)….3.2.1
Example: 5! = 5.4.4.3.2.1 = 120
Also, 0! = 1
Permutation:
The different arrangements of a given number of things by taking some or, all at a time are called
permutations.
 Arrangement refers to permutation.
Example: All permutations (or arrangements) made with the letters a, b and c by taking two at a time are
(ab, bc, ca, ac, cb, ba).

 Number of permutations: Number of all permutations of n things, taken r at a time is given by


n
Pr = n(n-1)(n-2)….(n-r+1) =n!/(n-r)!
Combination:
Each of different groups or selections which can be formed by taking some or all of a number of objects, is
a combination.
Example: Suppose we want to select two out of three boys A, B and C. Then possible selections are AB, BC
and CA. Note: AB and BA represent the same selection.

Number of Combination:
The number of all combination of n things taken r at a time is: nC = n!/r!(n-r)!
r

Some Important Facts:


 nC=1= n
0
nC

 nPr = r! nCr ; nPr = Arrangements of r things means first selecting r things and
arranging these.
 nCn-1 + nCr = n+1Cr
 nC0 + nC1 + nC2 + …..+ nCn = 2n
 nCn = 1
 nC0 = 1
 nCr = nCn-r
 If there are n objects of which p1 are alike of one kind, p2 are alike of another
kind and so on and pr are alike of rth kind such that (p1+p2+…pr)= n.
Then, number of permutations of these n objects is:
n!/(p1!p2!...pr!)
71
Determine if the situation represents a Permutation or a Combination:

Q1: In how many ways can five books be arranged on a book-shelf in the library?
A. Permutation B. Combination C. Both D. None of these

Q2: In how many ways can three student-council members be elected from five candidates?
A. Permutation B. Combination C. Both D. None of these

Q3: Seven students line up to sharpen their pencils.


A. Permutation B. Combination C. Both D. None of these

Q4: A DJ will play three CD choices from the 5 requests.


A. Permutation B. Combination C. Both D. None of these

Solution:
1. a 2. b 3. A 4. b

Permutation of things when some are identical

If out of n things, p are exactly alike of one kind, q exactly alike of second kind and r exactly alike of third
kind and rest are different, then the number of permutations of n things taken all at a time is
n!/p!*q!*r!

Example:
How many different words can be formed using letters of the word ARRANGEMENT?
Solution:
Total letters in the word are 11 and the identical letters are 2A, 2R, 2E and 2N.
So, total number of possible arrangements
= 11!/2! 2! 2! 2!

Example:
How many different words can be formed using letters of the word ARRANGEMENT if all the vowels come
together?
Solution:
Total letters in the word are 11. If we tie all the vowels together (2A, 2E), then we have 8 letters, out of
these 8 letters 2R, 2N are identical. These 8 letters can be arranged in 8!/2! 2! ways.
Now in a group of 4 vowels, 4 letters can be arranged themselves in 4!/ 2!2! Ways. So, total number of
words formed 8!/2!2! * 4!/2!2!.

72
Circular Permutation
Number of circular permutation of n different things taken all at a time = (n-1)!

In case of a necklace or garland as anticlockwise and clockwise arrangements are same. So, total number
of arrangements of n beads for forming a necklace = (n-1)!/2

Example: In how many ways can 7 boys be seated at a round table so that 2 particular boys are
a. next to each other b. separated
Solution: Let the 2 particular boys be taken together as one unit, then the number of units will be 6.
They can sit around the table in 5! Ways. And for each of these arrangements, 2 can be interchanged in 2!
Ways. Hence, total number of arrangements = 5! * 2!
b. The arrangements that the two persons are separated = 6! – 5!*2!.

Some Important Points:


1. Number of permutations of n different things taken at a time when repetition is allowed
= n*n*n*….r times = nr
Example: In how many ways can 4 rings be worn in the index, ring finger and middle finger, if there is no
restriction of the number of rings to be worn on any finger?
Solution: Each of 4 rings could be worn in 3 ways either on index, ring or middle finger.
So, 4 rings could be worn in 3*3*3*3 = 34

2. Number of selection of r things out of n identical things = 1


Example: In how many ways 5 marbles can be chosen out of 100 identical marbles?
Solution: It can be done in only 1 way because all the marbles are identical.

3. Total number of selection of zero or more things out of n identical things = n+1
Example: There are 4 identical oranges, 5 identical apples and 6 identical mangoes in a fruit basket. In how
many ways can a person make a selection of fruits from among the fruit in the basket?
Solution: Zero or more things can be selected in (4+1) = 5 ways
Likewise number of selections of apples and mangoes are 6 and 7 respectively.
But one of these selection number of each type of fruit is zero and hence this selection must be excluded.
Hence, required number of ways = 210 – 1 = 209

Total number of selections of zero or more things out of different thing = nC0 + nC1 + nC2 +…….+ nCn = 2n
and
The number of selections of 1 or more things out of n different things = nC1 + nC2 +…….+ nCn = 2n - 1

Example: Given 5 different green dyes, 4 different blue dyes, 3 different red dyes, how many
combinations of dyes can be chosen taking at least 1 green and 1 blue dye?
Solution: At least one green can be selected in 25 – 1 = 31 ways
At least one blue can be selected in 24 – 1 = 15 ways

73
At least one red or no red can be selected in 23 = 8 ways
Hence, total number of selections = 31*15*8 = 3720
Number of ways of dividing (m+n) different things in two groups containing m and n things respectively
= (m+n)!/m!n!

Example: In how many ways can 15 soldiers be divided into three groups of 8, 5 and 2 respectively??
Solution: Required number of ways = 15!/8!5!2!

Practice Questions

1. There are 3 questions in a question paper. If the questions have 4,3 and 2 solutions respectively, find
the total number of solutions.
2. How many 3-digit numbers can be formed with the digits 1,4,7,8 and 9 if the digits are not repeated?
3. Suppose you can travel from a place A to a place B by 3 buses, from place B to place C by 4 buses, from
place C to place D by 2 buses and from place D to place E by 3 buses. In how many ways can you travel
from A to E?
4. In how many ways can an animal trainer arrange 5 lions and 4 tigers in a row so that no two lions are
together?
5. There are 4 books on fairy tales, 5 novels and 3 plays. In how many ways can you arrange these so that
books on fairy tales are together, novels are together and plays are together and in the order, books on
fairy tales, novels and plays.
6. 12 points lie on a circle. How many cyclic quadrilaterals can be drawn by using these points?

1. 24 2. 60 3. 72 4. 2880 5. 17280 6. 495

74
Probability

What is Probability?
Probability is defined as the chance of happening or not happening of an event.
Example:
 In a toss of a single coin the chance of appearing a head.
 The chances of drawing a black ball from a bag containing two black and two red balls.

Types of Events
Simple Event or Elementary Event - An event is called a Simple Event if it is a singleton subset of
the sample space S.
Example: When a coin is tossed, then the sample space is S = {H, T}
Then A = {H} occurrence of head and B = {T} occurrence of tail are called Simple events.

Equally likely events - Outcomes are said to be equally likely when we have no reason to believe
that one is more likely to occur than the other.
Example: When an unbiased die is thrown all the six faces 1, 2, 3, 4, 5, 6 are equally likely to come up.

Exhaustive Events - are those which will cover all the possible outcomes.
Example: In a throw of a coin, getting a head or getting a tail are mutually exhaustive events because
they together can give whole sample space. P(A1 + A2 + A3 +……+An) = 1

Mutually Exclusive Events - Two or more events are said to be mutually exclusive if one of them
occurs, others cannot occur. Thus if two or more events are said to be mutually exclusive, if not two of
them can occur together. Hence, A1,A2,A3,…,An are mutually exclusive if and only if Ai∩Aj=ϕ, for i≠j
P(Ai ∩ Aj) = 0 for i≠j
Example: When a coin is tossed the event of occurrence of a head and the event of occurrence of a tail
are mutually exclusive events because we cannot have both head and tail at the same time

Mutually Exclusive and Exhaustive Events- Let A1, A2, …. , An are n mutually exclusive and
exhaustive events. Then, Probability of their unions is given by: P(A1 + A2 +…An) = 1
Example: Let us prove if for n =3, A1, A2 and A3
P(A1 + A2 + A3) = P(A1) + P (A2) + P(A3) – P(A1 ∩A2) –P(A2 ∩A3) – P(A1 ∩A3) + P(A1 ∩ A2 ∩ A3)
= 1 -0 = 1

75
Independent Events- Two or more event are said to be independent if occurrence or non-occurrence
of any of them does not affect the probability of occurrence of or non-occurrence of their events.
Example: Let bag contains 3 Red and 2 Black balls. Two balls are drawn one by one with replacement.
Let A is the event of occurrence of a red ball in first draw. B is the event of occurrence of a black ball in
second draw.

Then probability of occurrence of B has not been affected if A occurs before B. As the ball has been
replaced in the bag and once again we have to select one ball out of 5(3R + 2B) given balls for event B.

Example: Probability of a man staying alive after 20 years is 3/5 and that of his wife is 1/3. What are the
chances of only man staying alive after 20 years?
Solution: P(M) = 3/5 & P(W) = 1/3
P(only man) = P(M ∩W’) = P(M)P(W’) ( Independent Events)
= P(M){1-P(W)} = 3/5*2/3 = 2/5

Basic Axioms of Probability


Let S denote the sample space of a random experiment and E be any event
1. P(E)≥0 2. P(S)=1
3. For a finite or infinite sequence of disjoint events E1,E2,…P(E1∪E2∪E3∪…)=∑iP(Ei)
4. Let E' be the complement of E defined by E′=S−E. The following always holds: P(E)=1−P(E′)
5. P(ϕ)=0 6. If A⊆B ,then P(A)≤P(B) 7. 0≤P(E)≤1
8. For A and B, P(A∪B)=P(A)+P(B)−P(A∩B)
9. If A1,A2,A3……An are mutually independent events, then
P(A1∩A2∩…∩An)=P(A1)*P(A2)*…*P(An)

Practice Questions - Set A


1. An urn contains 6red, 5 blue and 2 green marbles. If 2 marbles are picked at random, what is
the probability that both are red? a.6/13 b. 5/26 c. 5/13 d. 7/26
2. A number X is chosen at random from the numbers -3, -2, -1, 0, 1, 2, 3. What is the probability
that |X|<2 a. 5/7 b. 3/7 c. 3/5 d. 1/3
3. A bag contains 21 toys numbered 1 to 21. A toy is drawn and then another toy is drawn without
replacement. Find the probability that both toys will show even numbers.
a. 5/21 b. 9/42 c. 11/42 d. 4/21

1. B 2. B 3. b

76
Odds in Favor/Against
If m be the number of ways in which an event occurs and n be the number of ways I which it does not
occur, then
a. odds in favour of the event = m/n ( or m:n)
b. odds against the event = n/m ( or n:m)

Example: The odds against an event are 5:3 and the odds in favour of another independent event are
7:5. Find the probability that at least one of the two events will occur.
a. 52/96 b. 69/96 c. 71/96 d. 13/96
Solution: c
Let probability of the first event taking place be A and probability of the second event taking place be B.
Then P(A) = 3 /(5+3) = 3/8 =>P(B)= 7 / (7+5) = 7/12
The required event cab be defined as that A takes place and B does not take place (A or B takes place
and A does not take place or A takes place and B takes place.)
=[P(A){1−P(B)}]+[P(B){1−P(A)}]+[P(A)P(B)]
=[(3/8)×(5/12)]+[(5/8)×(7/12)]+[(3/8)×(7/12)]
=(15+35+21)/96 => = 71/96

Note:
Let p be the probability of occurrence of an event in a random experiment.
Also let q = 1 – p
Then, probability of occurrence of the same event r times in n trials is given by nCr pr qn-r

Example: What are the chances of getting six sixes in 8 throws of a dice?
Solution: Let p denotes the probability of occurrence of a six in a throw of dice
Then, p = 1/6 => q = 1-p = 5/6
Here r = 6 and n = 8 ; So, answer is 8C6 (1/6)6(5/6)2 => = 700 (1/6)8

Conditional Probability
Let A and B be two events associated with a random experiment then the probability of occurrence of A
under the condition that B has already occurred and P(B) not = 0, is called the conditional probability
and it is denoted by P(A/B).
Thus, P(A/B) = probability of occurrence of B given that B has already happened.

Example: A bag contains 5 white and 8 black balls. Two successive drawing of three balls at a time are
made such that the balls are not replaced before the second draw. Find the probability that the first
draw gives 3 white balls and second draw gives 3 black balls.

77
Solution: Let A be the event of drawing 3 white balls in first draw and b be the event of drawing 3 black
balls. Hence, required probability = P(A ∩ B) = P(A) * P(B/A)
P(A) = 5C3 / 13C3 = 10/286 = 5/143
After drawing 3 white balls in first draw 10 balls are left in the bag, out of which 8 are black balls.
So, P(B/A) = 8C3 / 10C3 = 56/120 = 7/15
Hence, P(A ∩ B) = P(A)*P(B/A) = 5/143 * 7/15 => = 7/429

Practice Questions - Set B

1. If the chance that a vessel arrives safely at a port is 9/10 then what is the chance that out of 5 vessels
expected at least 4 will arrive safely?
a. 14×94/105 b. 15×95/104 c. 14×93/104 d. 14×96/105
2. An eight-digit telephone number consists of exactly two zeroes. One of the digits is repeated thrice.
Remaining three digits are all distinct. If the first three digits (from left to right) are 987, then find the
probability of having only one 9, one 8 and one 7 in the telephone number.
a. 1/18 b. 1/20 c. 1/10 d. 5/47
3. Two urns contain 5 white and 7 black balls and 3 white and 9 black balls respectively. One ball is
transferred to the second urn and then one ball is drawn from the second urn. Find the probability that
the first ball transferred is black, given that the ball drawn is black?
a. 13/23 b. 11/23 c. 14/23 d. 7/23
4. Dexter was born between October 6th and 10th (6th and 10th excluding). His year of birth is also
known. What is the probability of Dexter being born on a Saturday?
a. 0 or 1/3 b. 1/7 or 3/7 c. 1/3 or 1/7 d. Can’t be determined
Direction for Q5 - 7: One card is drawn from a pack of 52 cards, each of 52 cards being equally likely to
be drawn. Find the probability that
5. The card drawn is black
a. ½ b. ¼ c. 8/13 d. Can’t be determined
6. The card drawn is black and a queen.
a. 1/13 b.1/52 c. 1/26 d. 5/6
7. The card is either a king or a queen or a heart
a. 17/52 b. 21/52 c. 19/52 d. 9/26
8. A and B play a game where each is asked to select a number from 1 to 5. If the two numbers match,
both of them win a prize. The probability that they wit win a prize in a single trial is
a. 1/25 b. 24/25 c. 2/25 d. None of these

1. a 2. c 3. c 4. a 5. a 6. c 7. c 8. b

78
79
Review Exercise 13
Mensuration
# Questions: 10 Duration: 20 mints Negative Marking: 0.25 Calculator: Not Allowed

1. The length of a rectangular plot is 5(1/3), times that of its breadth. If the area of the plot is 270 square
metres, then what is its length?
a. 120 m b. 130 m c. 125 m d. None of these
2. The length, breadth and height of a cuboid are in the ratio 7: 5: 3 and its whole surface area is 27832 cm2.
Its volume is:
a. 28810 cm2 b. 28812 cm3 c. 288120 cm3 d. 288120 cm2
3. A room is 12(1/4) m long and 7 m wide. The maximum length of a square tile to fill the floor of the room
with whole number of tiles should be:
a. 175 cm b. 150 cm c. 200 cm d. 125 cm
4. The perimeter of a floor of a room is 17 m. What is the area of four walls of the room, if its height is 4 m?
a. 76 sq. m b. 48 sq. m c. 54 sq. m d. 68 sq. m
5. The circumference of the front wheel of a cart is 30 ft long and that of the back wheel is 36 ft long. What
is the distance travelled by the cart, when the front wheel has done five more revolutions than the rear
wheel?
a.20ft b.900 ft c.500 ft d.450 ft
6. Four horses are tethered at 4 corners of a square field of side 70 metres so that they can just about reach
one another. The area left ungrazed by the horses is:
a. 1050 sq.m b. 3850 sq.m c. 950 sq.m d. 1075 sq.m
7. A solid metal cylinder of 10 cm height and 14 cm diameter is melted and re-cast into two cones in the
proportion of 3 : 4 (volume), keeping the height 10 cm. What would be the percentage change in the flat
surface area before and after?
a. 9% b.16% c.25% d.50%
8. A circular road is constructed outside a square field. The perimeter of the square field is 200 ft. If the
width of the road is 7√2 ft. and cost of construction is Rs. 100 per sq. ft. Find the lowest possible cost to
construct 50% of the total road?
Let’s check your scores
a. Rs.70,400 b. Rs.1,25,400 c. Rs.1,40,800 d. Rs.2,35,400
9. Find the areanow!!!!
of the sector whose radius and central angle are 21 cm and 60° respectively?
a.231 cm2 b.400 cm2 c.600 cm2 d.750 cm2
10. Find the area of quad-rant with radius 64 cm?
a. 3218.28 cm² b.680.35 cm2 c.500 cm2 d.750 cm2
1a 2c 3a 4d 5b 6a 7d 8d 9a 10 a

1a 2c 3a 4d 5b 6a 7d 8d 9a 10 a

80
Review Exercise 14
Set Theory
# Questions: 4 Duration: 7 mints Negative Marking: 0.25 Calculator: Not Allowed

Directions for questions 1 - 2: Refer to the data below and answer the questions that follow.
In the Indian athletic squad sent to the Sydney Olympics, 21 athletes were in the triathlon team 26 were
in the pentathlon team and 29 were in the marathon team, 14 athletes can take part in triathlon and
pentathlon 12 can take part in marathon and triathlon 15 can take part in pentathlon and marathon and
8 can take part in all the three games.

1. How many players are there in all?


a. 35 b. 43 c. 49 d. None of these

2. How many were in the marathon team only?


a. 10 b. 14 c. 18 d. 15

3. Of the 200 candidates who were interviewed for a position at a call center, 100 had a two-wheeler, 70
had a credit card and 140 had a mobile phone. 40 of them had both, a two-wheeler and a credit card, 30
had both, a credit card and a mobile phone and 60 had both, a two wheeler and mobile phone and 10
had all three. How many candidates had none of the three?
a. 0 b. 20 c. 10 d. 18

4. In a class of 40 students, 12 enrolled for both English and German. 22 enrolled for German. If the
students of the class enrolled for at least one of the two subjects, then how many students enrolled for
only English and not German?
a. 30 b. 10 c. 18 d. 28

1. b 2. a 3. c 4. c

81

You might also like